You are on page 1of 466
MCAT is @ registered trademark of the Association of Amencar Medical Colleges, which does not endore this book Copyright © 2008 by Examkrackers, Inc All nights reserved under International and Pan-American Copyright Conventions Published in the United States of America by Osote Publishing, Lexington, KY 2nd Edition ISBN 1-893858-55-3, ISBN: 978-1-893858.55.8 To purchase additional copies of this ot any other book in the Examkrackers MCAT S-volume set, call 1-888-572-2536 or fax orders to 1-859-255-0109 cexamkrackers.com osote com audioosmosis.com Printed and bound sn the US A. [No part of this work may be reproduced for distribution to 2 third party m any form or by any means, electronic or ‘mechanical, including photocopying, recording, or any information retnieval system, without prior consent of the publisher, Osote Publishing. MCAT 1s a registered trademark of the Association of American Medical Colleges Copyright ® 2008 Examirackers, ne Invreobucron * wi \Versat Test 1 Versa Test 2 Versat Test 3. Versat Test 4 Versat Test 5. ‘Venwat Test 6 Veroat Test 7. Versa Test 8. Veroat Test 9, Versa. TEST 10... Veraat Test 11... ANSWERS & EXPLANATIONS TO VenBAt Test 1 Answers & EXPLANATIONS To VERBAL TEST 2 ANSWERS & EXPLANATIONS To VeRBaL TEST 3. ANSWERS & EXPLANATIONS TO VERBAL TEST 4 s.r ANSWERS & EXPLANATIONS TO VERBAL TEST Ss ANSWERS & EXPLANATIONS TO VERBAL TEST 6. ‘Answers & EXPLANATIONS TO VERBAL TEST 7. Answers & EXPLANATIONS To VERBAL Test 8 ANSWERS & EXPLANATIONS To VERBAL TEST 9 ANSWERS & EXPLANATIONS TO VERBAL TEST 10... Answers & EXPLANATIONS TO VeRsAt Test 11.. ANSWERS & EXPLANATIONS To VenBAt Test 12.. ANSWERS & EXPLANATIONS To Versat Test 13.. ANSWERS & EXPLANATIONS TO Versat. Test 14.. | Raw Score CONVERSION & ANSWER SHEETS... | Copright 2 2008 Esandrocker, le ‘vil © Exatecnacnes 101 Passaces in MCAT VeRvat REASONING Corynght 2008 Exarnkvackars Inc ‘Directions _ 1 Inemooucnon * 1 Famuharize yourself with the look, feel, and format of the first two Verbal Passages and questions. These appear just prior to Verbal Test 1. You are given 60 minutes for a full-length Verbal Reasoning test Take the three sample test passages frst, allotting yourself about 25 minutes. Grade your test and check the explanations for both correct and incorrect answers at the back of the book. . When you are ready to take your first full-length Verbal Test ensure that you will not be bothered for at least an hour. During the test, don’t answer the telephone, don’t sharpen your pencils, don’t stand stare out the window, and don’t get up to go to the bathroom. Treat these practice tests like the real thing It as not very practical or helpful to write your answers or considerations on a prece of notebook paper. It 15 always a good idea to mark up the multiple choice questions nthe fest tself a8 you go through them If‘A’ can’t be correct, then mark it off and go to ‘B’. If ‘B’ 1s possible, circle st, and go to ‘C’, and so on. That way you are at least eliminating and narrowing choices that are not possible or are less likely. Using the process of elimination is, a very helpful technique on the MCAT The Computer Based Test allows the use of strikethrough and highlight functions right on the screen to help with narrowing down choices. |. When you are done with your practice test, take a short break You've earned 1t. 5, Now grade your test. To find your raw score, count the number of questions that you answered correctly. Use the table at the back of the book fo translate your raw score into an MCAT scaled score How did you do? 6. Tum to the Answers and Explanations section of the book and examine the explanations Check your schedule for when you will take your next prachce test. Between tests you shouild also concentrate on the MCAT knoveledge-based areas of study Verbal Reasoning 1s a skills-based atea Practice will improve your © 7008 Fxarrarkes, nv x # ExanacrAcess 101 Passaces iN MCAT Veraat REASONING CConynght © 2008 Examkrackers, Inc Verbal Reasoning Warm-up (Three passages only!) Time: 25 Minutes Questions 1-17 2. © Exavernackers 101 Passaces in MCAT Veraal Reasonina VERBAL REASONING DIRECTIONS: There are three passages in this Verbal Reasoning warm-up test. Each passage 1s followed by several questions After reading a passage, select the ‘one best answer to each question If you are not certain of an answer, eliminate the alternatives that you know to be incorrect and then select an answer from the remaining alternatives Indicate your selection by blackening the ‘corresponding oval on your answer document Passage | (Questions 1-6) methods used fo teach acting skulls xtemal” and “internal” styles The “external” style, best exemplified by the London-based Royal Academy of Dramatic Arc (RADA), emphasizes the technical detail of 65 reproducing vowce, facial expression. and body language Actors trazned an this style are taught to mune the lassie external manifestations of emouions, sathout striving to feel the emouons themselves To this end, they study what are ‘commonly regarded as obvious physical indicators of the 10 emotional state being portrayed, and incorporate some or all of them into the role For example, actors, scientists, ‘and most of the public agree that lying is charaeterzzed by “shufly eyes.” avoudant eye contact, hesitant speech, elevated pitch, sweating, and/or hand wringing. ‘Thus, an “external” 15 actor portraying a har would consciously perform some or all of these motions whule reciting his lines, fo suggest to the audience the external traits they are hikely to associate with deception ‘Therein lies the pamary advantage of thiy classical, 20 “external” style since it conveys emotions through stylized. visible movements that the public mstinctively assocrates with identifiable feelings, it ususlly succeeds 1n letting audiences know which emotion the actor 18 trying. to portray Audiences are rarely confused, but there rsa risk the 26 performance may seem too overt. too melodramatic This usually happens when am overeager classical actor attempts 10 work in too many external indicators ofthe same emotion ‘The result 1s a character that obviously feels aa identifiable emotion, but seems (0 either feel st much more strongly than 30 the circumstances warrant orto be exaggerating his feelings. (0 the point of melodrama We should note that classical acung evolved on stage and bears the ampnnt of sts ongins. Classteal actors often practice Shakespearean productions, which affects their 96 style Since the Elvzabethan stage was unamplitied and viewed from a distance, classical acting. stressed. vocal strength, projection, and clear enuneration It also stressed general posture over small gestures and facial expression, since an Elizabethan audience could see only the former Copyright © 2008 Examirackes, nc 6 50 65 7 from their seats But, when this kind of acting 1s used 1m film, the actor may appear too rigid and theatncal, conducting @ street conversation with @ booming, stentonian vore and Oxford-precise enunciation This 18 not to say that classically trained actors have not made inroads into American film, consider Patrick Stewart, Jobn Gielgud, and Judy Dench However, you wall notice that such actors are usually typecast as commanding, digiified, well-educated characters, playing basicaly the same roles in every movie. ‘The other style of acting 1s the Stamislavsks Method, popularized im America during the 1930s at the prestigious New York-based Actors Studio and exemphfied by the young Marlon Brando. “Method” acting 1s an “internal discipline that aims to get the actor to actually feel the thoughts, motives, and emotions thatthe serip calls for the character to feel In Stanislavski's book An Actor Prepares, he utges actors to prepare for a role not by mumicking others’ mannerisms, but by taking time to contemplate the character’s circumstances and motivations, along with his purpose in interacting with the other characters, props, and his environment. ideally, this preparation lets the actor get into the mind” of the character and give a “natural,” ceffortess performance by “hving the life” of the character fon stage/sereen, Smee Method actors do not focus on ‘mechanical gestures, they give less mannered, more natural performance when at their best But Method acting also has nts drawbacks Furst t1s better sunted to the close-up camera than the stage, because Method actors’ rehance on facial expression and quieter, natural speaking styles will not project to the stage audience While Method acting aums for 2 subtler performance. ifthe actor does not accompany bis understanding of the character with overt actions suggesting ‘his emotions to the audience, his performance may be to0 subtle for the audience to apprectate GO ON TO THE NEXT PAGE. 1. According to the author, Patrick Stewart's success in the role as the commander of the starship Enterprise sn the sentes Star Trek was mn large part the result of his ‘A, previous roles in Shakespearean productions, B. _abshty to make inroads into American film C.__ previous classical training D. ability to incorporate some of Stanislavska’s Method. 2. On the basis of the passage, 1t 1s reasonable to conclude that A. Stamslavsks probably did not think much of the “external” style B. Marlon Brando would not have done well in an Elizabethan stage productos. C.anaaudience rmght be confused by a Method actor in wzabethan stage production 1 Grelgud was probably not capable of acting sung the Method 3. Assume that Judy Dench was interviewed. If Ms. Dench remarked thar often, in her film career, she had refused. roles after reading scnpts and realizing that she could not relate to What her character was feeling, this evidence would weaken the passage assertion that A. Ms, Dench was usually typecast, pl role every movie the RADA style enabled actors and actresses to mimic emotions C. the Stanrslavskt Method helps actors to get into the ‘minds of their characters D, Ms Dench was a classically wauned actress ying the same 4. Children may often he, or say that they will do something while having no intention of actually carrying through with what they are saying. What question might this nformation reasonably suggest abont the author's explanation of acting styles? A. Children can be good natural actors B. _Isn'tthe RADA style 2 more basic or natural style of aching? C,Aren’t most actors simply “lying” when they are 1m a film? 1D. Did Stonsslavsks analyze children to come up with the bass for hs style? Jaynght © 2008 Eearckars Ine Ver@at REASONING Waaw-ur © 3 5. According to the passage, the best actress to realistically portray the character of the blind Helen Keller in a television documentary would be one who has been trained prmanily in 1 the Method Tl the RADA syle TIL the “internal” style AL Tonly B. only C. Uonly D. Hand Ill only 6. According to passage information, when compared to Elzabethan stage characters, characters in films would appear AL more realistic B. larger than Ife G D, too subtle or dignified GO ON TO THE NEXT PAGE. 4 © Exaunnackees 101 Passaes in MCAT VeReaL REASONING Passage Il (Questions 7-12) It mught be argued that the main purpose of modera fashon 1s to make the wearer appear more altractwve, But, at least for men, that 18 not the main function that clothes ‘fulfil Fashion-onented clothing designers have indeed developed diverse methods to ereute the allusion of more perfect body proportions, and they are remarkably successful 1m conveying this image For example, verteal sinpes on a garment create an effective opbeal ilusion They make the 10 wearer himself appear more verucal—taller and thinner— thus improving the physical attractiveness of someone who 1s shorter and/or heavier than average On a jacket, padded shoulders convey the appearance of muscular shoulders and arms, while a narrow, tapered wast shows off (or creates 18 the allusion of) tm, athleuc abdomimals Sumalary, a ‘gament’s color, seem juxtaposed against the wearer's skin, affects viewers’ perception of skin tone For example, pale skin seens sallow against a dark, navy blue suit, bat tanner with a light start and jacket In high fashion, color aums to 20 mimic the color of the wearer's hair andor skan undertone, for a more harmonious blend Again, we should emphasize that these effects are not just overblown claims by clothes peddlers; clothing an the fight colors, pattems, and cut will actually make its owner appear more ideally proportioned 25 Yet the best-designed, most form-flattering clothes are not necessarily the most popular or acceptable ‘Tradinon played a greater role than any other influence am men’s clothing [nthe Middle Ages (and probably earlier), clothing was strictly regulated by law, certain colors could 30 be worn only by the upper classes or by certain guilds, on penalty of a fine Since that time, clothing has become a uniform of elass, rank, and calling, which has endured as 2 “dtess code” long after the legal rules were repealed While the look of “power” clothiag has certainly changed, there 385 remains a “power” look, which 1s expected dress for those 1m power or aspiring tout Currently, the “power” look is the British business sutt, wath ats Limited range of dark colors and simple patterns Simply put, executives are expected to wear a power surt, 40 and the mere Wearng of such suis signals subconsciously to viewers that the wearer must be a wealthy and powerful businessman. ‘The power look gravitated toward the Brush heavy woolen suit because of a historical accident, Enghshmen were dnven by combinauon of ingore, 45 greed, and gunpowder to become the dominant colonial empire, and thus exported to economically and culturally subyugated nations their dismal fashion sense Note that English fashion 1s not badly susted to hfe n the British Isles The cold, sodden chmate of England and Ireland are well 50 served by heavy woolen suits and hats and heavy brogue shoes. However, these items are not suitable for export to the sweltering dry heat of Africa and the antebellum South. Copyneht © 7008 Examkerrkes, lnc ‘or humid summers on New York’s Wall Street And yet these ancient uniforms persist throughout all of Britain's former 5 colomes despite thew obvious impracuicality Many stems of clothing are worn purely for traditional purposes For example, the ue has no rattonal function that anyone can explain, yet most businessmeu agree thatthe te 18 a man’s most important accessory. The reasons are again 60 historical The ue evolved from other, more archaic badges Of class und rank, whose symbolic ymplications persist. The traditional, dragonally striped “repp” nie 1s more properly called the “regimental” pattern, which was once worn by British Army officers to symbole their high status and 65 access to guns Sumularly, the repesting-pattemed “foular evolved from ancient heraldic crests, which medieval anstocrats once wore on their shields and pennants to symbolize their connection to wealthy, landowning, arms- bearing clans Even more recently, the “Ivy League” tie 70 pattern was worn by wealthy East Coast scions to symbolize thetr elite schooling and connection to the “old boy” power network ‘Unfortunately for fashion, these traditions are so deep- seated that they cannot be changed in one Inenme by the 75 average person, of even by vast adverusing campaigns by designers While a businessman would look objectively “better” by eschewing the traditional “red tie, white shirt, blue suit” uniform sn favor oF a color, cut, and pattera more suited to his individual body, doing so Would be unwise 80 Research shows that a umiform’s historical umplicattons, ‘rump any aesthetic considerations and have much more of ‘an umpact on the viewer Thus, conformuty produces the best results, Hot the best fashions 7. The author most likely beleves that one of the main purposes of clothng should be 10 provide the wearer wath A. apower” appearance. B. aumform C. amore attractive appearance D. an appearance appropriate to bis employment GO ON TO THE NEXT PAGE, 8. The author's charactervation of Britain suggests that che retorvicomment that the “power look gravitated toward the British heavy woolen suit because of ahistorical accident” (ine 42-43) meant that: the author believes the former colomies could have defeated their British subyugators the author believes the Bnush business suit 1s appropnate for the Baush Isles the author does not think much of Briain the author does not think much of the British 9. Amappropnate theory based upon the emerging dominance of the British business suit (lines 37-5) 18 that the best designed clothes are not necessanly the most comfortable. the traditional Bash ‘power’ Took has even permeated the American business landscape, people will go to great lengths 1m their efforts to tnerease theur status people usually don't dress appropnately for thew prevaslng climate 10. The “expected” business suit jacket mvanably has four seemingly useless buttons sewn on the cuff of each sleeve TThe information presented on the origins of fashion makes ‘which of the following ideas most plausible? A. Cooyegh ‘The extra buttons sewa on the sleeves actually serve to create and enhance the appearance of strength and authonty 1a man's hands ‘The appearance ofthe “extra” buttons 18 a relatively new adelition to jackets and 1s believed to somehow render the wearer more atracuve. ‘Tradionally four buttons have always been sewn con the sleeve of yackots and have never served any usefal purpose Tradiuonally, oyster-shell butions were so susceptible 0 breaking that “extras” were always sewn on the sleeve 8 Examkrackers I Versat REASONING WaRw-ur # 5 11. The amthor implies that the primary purpose of women's 2 fashion 1s: A. to enhance thetr appearance. B. tobe more practical C. to promote the “power” look, but in a different D. toconfront radiuonal styles ‘The author probably mentions that “conformty produces the best results, if not the best fashions” (82-83) 1n order wo demonstrate designers support the clam that there is an expected “look” alastrate the persistence of beliefs that confhit wath the demands of the business world provide an example of the application of traditional clothing the weakness of some fashion GO ON TO THE NEXT PAGE. 6 © Examneackers 101 Passages im MCAT VeRaAL REASONING: Passage Ill (Questions 13-18) Whale Machiavelh’s The Prince 1s often hatled as the first breakaway from classical (Greco-Roman) pohtucal theory toward the shrewder and more pragmatic modern form, there 1s clear evidence that his system, “realpohtuk”— 5 politics without principles—was practiced extensively long. before hus day, though 1 was genteelly onnited from popular politcal teauses ‘One notably evil predecessor to Machiavelli was the ‘early Spanish mpersal advisor Ortega Sorolla, a prototypical 10 represemtatve of Spain's early Legahist school Sorolla ‘openly advocated the perpetuation of the ruling emperor bby any means, going so far as to recommend thatthe state execute men of exceptional talent who mught one day aspite to the throne 18 For Machiavellians, power 15 tangible and material the strength of the army that a state can muster 1s at once the mamfestauon and measure of us power Machiavellt considers as strong only those “who can, through abundance ‘of men or money, put together a sufficient army, and hold the 20 ficld against anyone who assails them” Those who cannot must bargain or assume the delensive, and those that are «disarmed become the targets of aggression and contempt For Machiavelli, the price of ruling 1s eternal vigilance and constant warfare, he urges aspirants to power 10 “have 20 25 other aum or thought but war the chief cause of the loss Cf states, 1s the contempt ofthis art.” ts not unl Chapter XIT that Machiavell: mentions for the first tame that 2 state must also have good laws, but seems so much like an afterthought that the laws of qualty 30 seemingly nieans litle to Machavellt For one thing, he never defines what qualities make the kaw * good,” and his Inte statement that “where there are good arms there must be good laws” 1s unsupportable Thus, for Machiavelli, the quality of government, and of runtcrpal Iie under 1, 25 appears unimportant ‘Machiavelli holds the view that the prince does what hhe does 1 order to gain the glory of a good reputation, even af posthumously He gives an adminng account of Agathocles’s rise from poverty to kingship of Syracuse 40 by seizing control of the army and butchering the proper senators and eminent citizens, he gained great power and apparently lived out his natural life. Yet this 1s sill noe the Mactnavellian ideal, he comments," by these methods fone may indeed gain power, but not glory his countless 4 atrocities .. do not permut of ius being named among the ‘most famous men” Yet if a ruler’s self-evaluation un life must include how others will remember hum after death, how cam any would-be prince rest easy, knowing that if he 18 defeated, history can be rewritten, and no one will be able to 50 know what his character vas truly like? oynight 82008 Fxemkracters, Inc ‘The ultimate embodiment of realpolitik might well be the totalitarian communist government envisioned in George Orwell's 1984 In his vision of a future England controlled by communsm, he depicts a ruling clque—the Inner 55 Party—which controls the populace through propaganda and “disappearances,” by encouraging ractal violence and hysteria, and by waging never-ending war with ts neighbors simply to destroy surplus wealth, thus keeping the population at subsistence level Since the propaganda ministries control 60 all nformation, they conteol the things postenty will say of them, hence their motto “Who controls the past controls the future, who controls the present controls the past” At fone point, an Inner Party member lays bare the principles of the Party, which illustrates the possible consequences of 6 realpolink’s extical flaw We are not interested [he says] in the good of others. not wealth or luxury or long life or happiness; only power . the German Nazis and the Russian Communists pretended, that they had 70 seized) power unwillingly and for a imuted ume, ‘and that yust around the corner there lay a paradise where human begs would be free and equal. We are not like that. We know no one ever serzes power with the intention of relinquishing xt. Power 75 means; tis an end.” 13, According to the passage, a ruler's power would be determined by measuring A. the qualities of his laws and his renown B. Ins renown and his abulty as a strategist. C._the strength of his army and if he was contempuble. D. the strength of his army and the qualues of ts laws GO ON TO THE NEXT PAGE. 14, Given the fact that George Orwell wrote 1984 after World War Il, long after the death of Machiavellt, we may assume, based on passage information, that. 18, 16. Conyrigh © 2008 Orwell admured the Nazvs. ‘Orwell was saurzang Machiavellian ideas Machuavellt admired the Nazas. (Orwell admured Machuavetl: According 10 the passage, Machiavelli believed that the tulumate goal of ruler is ‘ternal vigilance and constant warfare. power the strength of bis army, hus legacy Elsewhere im The Prance, Machiavelli wrote, “States that mise quickly, just as all the other things of nature that are bom and grow rapidly, cannot have roots and ramuticattons, the first bad weather kalls thern "This ost strongly challenges passage descriptions of: the Inner Party Agathocles the totalitarian communist government in 2984 ‘Spain's early Legalist schoo! nteackers Ine Verea. Reasoning Wanw-ur #7 17, According to the author, Machiavellh would have considered all of the following to be examples of power EXCEPT A, extreme loyalty to the ruler B. strong army C. money D. powerful weapons ‘STOP IF YOU FINISH BEFORE TIME !S CALLED, CHECK YOUR WORK YOU MAY GO BACK TO ANY QUESTION IN THIS TEST. STOP. 8 © Exannackers 101 Passaces IN MCAT VeRBaL REASONING Verbal Reasoning Warm-up Answers & Explanations WARM-UP ANSWER KEY 10 Examcaccers 107 Passages In MCAT VERBAL REASONING Passage | (Questions 1-6) 1. According (o the author, Patrick Stewart’ success in the role as the commander of the starship Enterprise in the senes Star Trek was mn large part the result of his A previous roles in Shakespearean productions WRONG It 1s accurate that Stewart had “previous. classical training” Some “classically trained actors hhave made inroads into American film, consider Patrick Stewart ~ (lines 45-46), However, itis 2 ‘major assumpiion that he had any previous roles un Shakespearean productions absuy fo make mmroads unto American film, WRONG This seems merely to be regurgutating passage information There 1S no real cause-and- effect relationship between this answer and Stewart's success mn the role as the commander previous classical craining CORRECT: It is accurate that Stewart had previous classical training” Some “classically ‘tamed actors have made inroads into America fil, consider Patrick Stewart” (lines 4546), Further, Patrick Stewart's role as “commander” (regardless of whether you know anything about Star Trek or not) was probably a success because "such actors are usually typecast a commanding, dignified, well educated characters” (lines 47-49) ability to meorporate some of Stanislavski's Method WRONG There 1s indication that Patrick Stewart used or was even avvare of the Stamislavskt Method This 1s pure conjecture 2. On the basts of the passage, it 1s reasonable to conchide that, AL (Copyright © 2006 Examare Stamslaysks probably did not think much of the “extemal” style WRONG This1snot a reasonable conclusion. Other than the man’s name, there sno spectfie information, ‘on Stanislavskt or what he thought Marlon Brando would not have done well in an Elizabethan stage production WRONG This 18 not a reasonable conclusion We can assume that since “the Elizabethan stage was ‘unampliied and viewed from a distance . [doing well in an Elizabethan stage production required) vocal strength, projection, and clear enunciation” (ines 36-38) However, it 1s another assumption altogether that Brando had no training or background 1m this area and would not have done well an audience imght be confused by a Method actor in ‘an Elizabethan stage production CORRECT: This 1s 2 reasonable conclusion Nowe the word might ‘The rncluston of this ‘softener’ opens up the possibihties in this answer Because the “Ehzabethan stage was unamphfied and viewed from a distance. [doing well 19 ‘an Elzahethan stage production required] vocal strength, projection, and clear enunciation” (Imes 36-38) And because the “external” style “conveys emouons through stylized, visible movements that the pubhe mstinctvely associates with wlentifiable feelings, 1 usually succeeds an Ieiting audiences know which emotion the actor 1 trying 10 portray Audiences are rarely confused” (lines 20-24). In contrast to the external style 1s the Method. “While Method acting aus fora subtler performance, the actor does not accompany his understanding of the character with overt actions suggesting his emotions, to the audience, his performance may be too subtle for the audience to appreciate” (lines 73-75). This ‘might confuse an audience John Gielgud was probably not capable of acting, using the Method. WRONG Thiss nota reasonsible conclusion There 18 no way of knowing this. Ibis pure conjecture 3. Assume that Judy Dench was interviewed, If My Dench remarked that often, n her film caret, she had refused roles after reading scripts and realizing that she could not relate to what her eharacter was feeling, this evidence would weaken the passage assertion that The correct answer must satisy two enteria The first 1» that it be ‘accurate passage asseruion In this question, all of the answer choices are accurate passage assertions Be aware, this 15 nor always the case on the MCAT The question 1s nar telling you to assume or trust that all of the following assertions are accurate You must know The second criterion 1s that the assertion be weakened by the formation in the assumption Ms Dench was usually typecast, playing the same role m every move WRONG This 1s an accurate passage assertion However, 115 nor clearly weakened by the passage ascumpuon. For stance, what af the roles. she refused were all pretty much the same? There 1s no ‘way of knowing and this 1s not the best answer. the RADA style enabled actors and actresses to ‘mimic emotions. WRONG This 1s an accurate passage assertion. However, 11s nor clearly weakened by the question assumption Apparently, Ms. Dench did not decline the roles because she was unable to “mime emotions,” but because “she could not relate to what her character was [actually] feeling.” C. the Stanislavskt Method helps actors to get into the ‘munds of their characters WRONG This 15 an accurate passage assertion However, it 18 not weakened by the passage ‘assumption because Ms. Dench was considered a classically tramed actress (lines 44-47) D. Ms Dench was a classically tramed actress CORRECT: This 15 an accurate assertion (lines 44-47), which 1s weakened by the assumption 19 the question The author used her as one of three ‘examples ofa classically trained actor, and therefore ‘one can assume that she was probably predominantly fan actress in this style However, refusing roles because “she could not relate to what her character was feeling” 1s not characteristic of the classical style This shouldn't have mattered toher She would ‘only be “mimicking” her characters overt actions It was Stanislavski’s Method actors who would have been concemed with actual feelmgs (line 56) The assumption would rndicate that Ms Dench was: also unfluenced to a greater or lesser degree by the Method 4. Children may often hie or say that they wall do something while having no intenuon of actually carrying through with what they are saying What question might this snformation reasonably suggest about the author's explanation of acting styles? ‘A. Chulden can be good natural actors WRONG The correct answer requires a “question” ‘This answer 1s nor a “question.” B,_Isn'tthe RADA stylea more bastc or natural style of acting? CORRECT: This answer 1s reasonably suggested by the information The London-based Royal Academy of Dramatic Art (RADA) style teaches actors “fo mimic the classic external manifestations ‘of emotions, without striving 10 feel the emotion themselves” (lines 6-8). Contrast this style with the Method style an whch actors are taught 10 ‘actually feet” the characters emouons, desires, and motivations. The children inthe question are clearly exhibiting the RADA style of acting: acting/saying. fone thing, while thinking/feeling another They are children-in-general (there 1s no specificity), and 118 Togical to assume that most of them have not had any formal acung traming, but are doing this “naturally” asa ‘basic’ way of behavin C. Aren't most actors simply “lying” when they are an a film’? WRONG This answer 16 nor reasonably suggested by the information It 1s possible that ths answer had been more specific mn referring to RADA actors ‘only, then it would have been reasonable However, “most” actors 18 4 generalization and severely limts Copyright ©2008 Exomtrackers, Inc Versa Reasonains Wastin the ‘rightness’ ofthis answer Notice that this answer even gives you a hint regarding who 111s referring to with, “when they are un a film” Thus answer 1s referring (othe Stanislavsk-style actors Thus, there 1$no companson or relanonshup with the “children” an the question D. Did Stamslavski analyze children to come up with, the basis for his style” WRONG: This question is nor reasonably suggested by the information ‘The “children” in the question are (probably unconsciously) using the RADA style “to mmc the classic external mantfestations ‘of emotions, without striving to feel the emotions themselves” (lines 6-8) Thus, there:snocompanson (or relationship withthe “children” mn the question. 5. According to the passage, the best actress to realistically portray the character of the blind Helen Keller m a feleviston documentary would be one who has been tramed primarily in L._the Method. WRONG This choice asthe same as Ill An actress primarily trained in this style would nor be the Dest choice Stanislavskr’s Method “urges actors to prepare for a role not by: mumacking others mannerisms” (lines 58-59) Yet the question suggests that we want someone who can “nme Helen Keller as closely as possible HL the RADA style. CORRECT: An actress trained promarily in this style would be the best choice The fact that Helen Keller 1s blind” ts of no consequence to the answer. ‘The fact that she will be portrayed in a “television documentary” 1s misleading because of the similanty to “film ‘The key to this question 1s the realisuc portrayal There 1s no suggestion here of interpreting emotions, morwations, circumstances Stanislavskr's style “Urges actors to prepare for a role nor by mimicking others’ mannerisms” (lines 58-59) Yet the question suggests that we want someone who can “munuc” Helen Keller as closely as possible The “London-based Royal Academy of Dramatic Art (RADA), emphasizes the technical detail of reproducing vorce, factal expression, and body language” (lines 3-5) the ‘interna’ style WRONG: This chowe 1s the same as 1 An actress pronariy trained in this style would not be the best choice Stanislavski's anternal style “urges actors (0 prepare for a role nor by mmicking others’ mannerisms” (hnes 58-59). Yet the question suggests that we wanr someone who can “mumic” Helen Keller as closely as possible 12» Examneackers 101 Passaces In MCAT Veesat Reasons A B. c D. Toaly only CORRECT Monly Wand M only See the explanation above 6 According to passage information, when compared to Elvzabethan stage characters, characters in films would appear A. more reahstie, CORRECT: The question does not say phrase “would sometimes appear” or “could appear” The ‘question says, “would appear.” The reference to “characters in films” clearly refers to actors using the Stamslavskt Method “Since Method actors do not focus on mechanical gestures, they give a less mannered, more natural performance when at thesr best” (lines 66-68). In contrast, “Since the Elizabethan stage was unamplified and viewed from 2 distance, classical acting stressed vocal strength, projection, and clear enunesation But, when thus kind of acting 1s used an film, the actor may appear too ngid and theatncal, conducting a street conversation with a booming, stentorian voice and Oxford-precise enunciation” (lines 3624), “larger than Ife” WRONG It is the Elizabethan characters who would have appeared “larger than life’ “Since the Elizabethan stage was unamphified and viewed from a distance, classical acting stressed vocal strength, projection, and clear enunciation — But, when tt 1s used in film, the actor may appear too ngxd and theatncal, conducting @ street conversation with @ booming, stentonan voice and Oxford-precise ‘enunetation” (ines 36-44) The phrase “charactersin films” clearly refers to actors using the Stanislavskt Method. 8 commanding or digesfied WRONG It was the “classically traned actors.” whose ongins can be traced back to Elizabethan productions. who were “usually typecast as commanding, dignified, well-educated character (ines 48-49) The phrase “characters in films” clearly refers 10 actors using the Stamslavski Method too subtle WRONG: The question does not say “would somenmes appeat” or “could appear” The question says, “would appear” Ia Method actor does not do weil, then “his performance may be 100 subtle” (line 76) However, they are not always too subile This 1s ot the best answer Copyright @ 2008 Exomrachers, Ine Passage Il (Questions 7-12) 7. The author most likely believes that one of the main purposes of clothing should be to provide the wearer wath A. “power” appearance WRONG: The author 15 not likely to beheve this ““Unforumately for fashion, these tradiwons [Le clothing worn for no rational purpose other than to emulate ‘power’ or “authonty’) are so deep- seated that they cannot be changed in one Iifetume by the average person, or even by vast advertising campaigns by designers” (lines 78-81). uniform, WRONG: The author 18 not hkely to believe this ‘Unfortunately for fashion, these traditions are 0 deep-seated that they cannot be changed in one lfeume by the average person, or even by vast adverusing campaigns by designers” (lines 78-81) C. amore attractive appearance. CORRECT: Though the author 1s a ‘reahst’, he 1s Likely to believe this Consider the disparaging way that he desenibes the rising dominance of the British business sutt (lines 40-59) The author clearly has 4 preference for “fashion” And, “Fashion-oriented clothing designers have indeed developed diverse methods t0 create the illusion of more perfect bod; proporuons [1e a “more altractive appearance” (ines 5-7). Consider the author's statement, “Unfortunately for fashion, these traditions are so deep-seated that they cannot be changed in one lifetime by the average person, or even by vast adverusing campaugns by designers” (lines 78-81), 1D. an appearance appropriate to his employmer WRONG The author 1s not likely to believe this, Though the author does apparently believe in comfort over the waditional ‘powers’ look (lines 54-59), there are no other examples. regarding. other types of employment which would allow us to draw this conclusion {fthe author had his way, the emphasis would be on snaking a person appear ‘The author's charactervation of Britain suggests that the retort/comment thatthe "power look gravitated toward the Brinsh heavy woolen suit because of a historical accident” (ines 45-47) meant that A. the author believes the former colonies could have defeated their Bniish subjugators WRONG This 1s not the suggested meaning This answer 1s obviously on the right track, but too speculate. There 1s no support for the speculation We don’t specifically know why the author believes. that the Brush becoming “the dominant colonial empire” was an “aceident” other than that he doesn’ ‘seem to think much of them (Answer D) B. the author believes the British business suit 1s apptopnate for the Brush Isles WRONG: This answer may be attractive because as an accurate statement. However, 1s not the suggested meamng of the author's comment ‘This answer seems to have nothing to do with the comment C. the author does not think much of Bntain WRONG This answer may be attractive because 1 is an accurate statement ‘The author's description of the “cold, sodden climate of England and Ireland’ (ines $2-53) indicates that this 18 not his favonte vacation spot However, this really has nothing to do. with the comment. the author does not think much of the British CORRECT: This 1 the suggested meaning This encompasses the possibibes of Answers A and CC. We know that the author does not think much of the British from the terms he uses to describe them: “dismal fashion sense,” “yingoism, greed” ‘The author would, in 2 Way unrevealed to us an the passage, argue that 1¢ was an “accident” that the Brush were able to “become the dominant colonial cempure™ and transfer their “dismal fashion sense” ‘Ths asthe best answer chorce. 9. Amappropnate theory based upon the emerging dommance of the Betish business suit (lines 40-59) 1s that A. the bestelesigned clothes are not necessanly the most comfortable WRONG: This 1s not an appropnate theory. The author does not imply that the Bash business sunt falls into the category of “the best-designed clothes.” the traditional Brush “power” look has even permeated the American business landscape WRONG, This 1s not an appropriate theory because Ls not accurate, British business susts “are not suitable for export to .. the antebellum South, oF to the hurd summers on New York's Wall Street ‘And yet these ancient uniforms persist shroughout all of Brtain’s former colonies despite their obvious. smpracticalnty” (lines 54-$9), C. people will go to great lengths sm their efforts 10 Increase their status CORRECT: This 1s an appropriate theory. It 1s clear that Wearing a “power” suit 1S a necessity for amereasing one’s status. ‘The current “power” look 4s the British business suit “And yet these ancient ‘uniforms persist throughout all of Britamn’s former colomes, despite their obvious impracticality” (Imes 57-9) Further examples of clothing items that Copprght © 2008 Exarneraceers ne ‘VerBal REASONING Waaw-uP © 13 have “no rational funewon" yet are replete m the passage (lines 60-77) mportant” are people usually don’t dress appropnately for ther prevailing climate WRONG: This 1s not an appropriate theory If we changed “usually” 10 “sometimes” this answer would have a chance As it stands, i not accurate Clearly, “The cold, sodden climate of England and Ireland are well served by heavy woolen suits and hhats and heavy brogue shoes” (lines 52-54). 10, ‘The “expected” business suit jacket mvanably has four seemingly useless buttons sewn on the euffof each sleeve ‘The information presented on the onigins of fashion makes ‘which of the following teas most plausible? ‘Thusiszcommontype ofsupposiion question that provides {you wath information from outside of the passage You are then expected to extrapolate on the theories or ideas presented by the author. Do nor get hung up or annoyed if you believe that the extra mnformation or supposition 1s, snaccurate, It obviously does not matter Choose the best A ‘The extra butions sewn on the sleeves actually serve to create and enhance the appearance of strength ‘and authonty m a man's hands, WRONG: This idea 1s not the most plausible The reference to “strength and authority" 1s an enticing aspect of ths answer However, “enhancing the appearance” 1s a “fashton-onented” aspect of clothing, which 1s subservent to “uadition. Particularly when the new information specifies “the expected business suit jacket” ‘The appearance of the extra buttons 1s a relatively new addition to jackets and 1s believed to somehow sender the wearer more altracuve, WRONG. This idea 1s not the most plausible The crux of the passage emphasizes, “tradition played 4 greater role than any other influence 10 men's clotiung” (lines 29-30) “Relatively new addition 1s mm conflict wath this Further, “sender the wearer more attractive” 1s a “fashron-oniented” of clothing, which 1 subservwent to“ Particularly when the new information specifies “the “expected” business suit jacket.” ‘Trodinonaly, four buttons have always been sewn ‘on the sleeve of jackets and have never served any useful purpose WRONG. Thus idea 1s not the most plausible This answer 1s attractive because 1t incorporates “wadinon” and alludes to serving “no rational function” However, the author does not say that twaditional tems of clothing have “never served any useful purpose” On the contrary, 1t seems that the ‘purpose of what now seems to serve “no rational 1d © Examwracers 101 Passaces in MCAT Versat REasonins function” can rather invariably be traced back 10 Ms original purpose, such as “guild” designations, ‘warmth, uniforms, and badges (ines 29-39, 52-54, and 60-78). Traditionally, oyster-shell buttons were $0 ssuscepuble to breaking that extras were always sewn, con the sleeve CORRECT: This idea 1s the most plausible This answer incorporates. “tradition,” which “played ‘2 greater role than any other influence in men’s clothing” (Innes 29-30) Additionally, 1n contrast to Answer C, it provides a rational explanation for why there were four buttons. Best of all, the traditional reasoning for having the extra “oyster-shell” buttons, serves “no rational function” today 11, ‘The author umphes that the primary purpose of women’s fashions AL to eabance thew appearance CORRECT: This ss plied by the author fn other words, the author implies that “the main purpose ‘of modern fashion 1s to make women appear more altractive "The absence of any mention of “women” 1m the passage 1S conspicuous. However, the caveat 1m the second sentence obviously refers to women “It might be argued that the main purpose of modern fashion 1s to make the wearer appear more atractwe. But, at feast for men, that § not the main function that clothes fulfil” (Lines 1-4) to he more practical WRONG This 1s not imphed by the author See Answer A. to promote the “power” look, bur im a different WRONG. This 15 not 1 Answer A plied by the author. See « confront traditional styles. WRONG. This 1s not implied by the author There 1$.no way of knowing what women’s “traditional styles” would consist of 12, The author probably mentions that “confornty produces the best results, if not the best fashions” (lines 87-88) 1 order to, A, demonstrate the weakness of some fashior designers WRONG This 1s not the reason For one thing, the author apparently thinks that fashion designers are very effective at what they do “Again, we should emphasize these effects are not just overblow clams by clothes peddlers, clothing in the right colors, patterns, and cut wall actually make ts owner appear more xdeally proportioned!” (lines 22-26) Conynight 6 2008 Examkrackers, fe ‘support the claim that there 1s an expected “look CORRECT: This 1s the reason We know that the author beleves that there 15 an “expected look” im the business world, That 1s the “conformity” be refers to in the question Further, the quote in the question does support the claim in this answer sllustrate the persistence of beliefs that conflict with the demands of the business world WRONG: This 15 not the reason This answer 1s very muddled, and doesn’t make much sense The ‘only demand of the business world 1s to have the “power” look, Who has these persistent beliefs? The author? Certainly not the millions of businessmen ‘who are wearing business suns, It seems that only “designers” and the “average person” have these “persistent beliefs” This 18 not the best answer provide an example of the application of traditional clodung WRONG: This 1s not the reason. The quote in the {question is nor an “example” Examples are provided from lnes 60-77 Passage Ill (Questions 13-18) 13, Accordmg to the passage, a ruler’s power would be determined by measuring: A c the qualities of has laws and hus renown WRONG The “qualities of his faws” would not be ‘a determiner of a ruler’s power. “the law of quality seemingly means Ittle to Machiavelli” (lines 30— 31). ts renown and his abilty as a strategist WRONG: These would not be determiners of a tuler’s power. Beyond there being no menuon of “ability as a strategist” there 1s a0 implication of this Aa “abihty” would not be “tangible and material", “For Machavellians, power 15 tangible ‘and matenal” (line 15). though tus 15 rendered ‘somewhat ambiguous in the passage by references to “contempt.” ‘the strength of hus army and if he was contemptible CORRECT: Both of these qualities are among those that could be “measured” to determine power, “the strength of the army that a tate can muster 1s at once the manifestation and measure of us power” (lanes 16-17), and “those that are disarmed become the targets of aggression and contempt” (lines 22-23). the swength of his army and the qualines of his laws WRONG: The “qualiues of his laws” would not be a determiner of a ruler’s power, “the laws of quality seemmngly means litle to Machavelh” (lines 30 3D 14, Given the fact that George Orwell wrote 1984 after World War Il, long after the death of Machravelh, we may assume, based on passage information, that A. Orwell admited the Nazss WRONG This 1s not a vahd assumption The extensive quote, which 1s an Inner Party member's soliloquy (lines 69-79) ashe “lays bare the prinesples of the Party” (hne 66) revels. according 10 the author, “realpoliuk’s eruucal flaw” (Line 68) Orwell, ‘who wrote the book and the soliloquy, would have been unlikely to have revealed any faws ina system which cantcatured the Nazis rf he so admired them ‘Orwell was satnzing Machiavellian ideas CORRECT: This 1s the only vabd assumpuon. The extensive quote, which 1s an Inner Party member's solsloquy (lines 69-79) ashe “lays bare the principles of the Pary* (line 66) reveals. according © the author, “realpolitik’s crueal law” (line 68) Orwell, who wrote the book and the sobloguy, would have been unlikely to reveal am flaws in Machiavellian sdeas in dhs fashion af he had been doing anyttng other than “sanizing” or making fun of them C. Machiavellt admired the Nazis WRONG This 1 not a valid assumption If, as provided 1n the question, Machiavell died long. before Orwell und WWIL, then the Nazis would not have existed 1m his lifetime for hum to have ever known of admired them D. Orwell admired Machuavelh WRONG This 1s not a valid assumption The extensive quote, which 1S an Inner Party member's soltloquy (lines 69-79) ashe “lays bare the principles of the Party” (Ine 66) reveals, according to the ‘author, “realpolink’s crsucal flan (lane 68) Orwell, who wrote the book and the soliloquy, would have been unlikely to have revealed any flaws in a system which cancatured Machravell, among others, sf he so admired 15, According to the passage, Machavellt believed that the ulumate goal of a ruler is A. elemal vigilance and constant warfare WRONG This was not 2 goal but a “price” to be paid. “For Machiavelhy the price of ruling 1s eternal vigilance and constant warfare” (lunes 23-24) B. power WRONG: This was not Machiavelh’s goal ‘goal of the Inner Party in 1984 (lines 78-79). For Machnavellt, power was not a goal, un and of itself {La means by which to achieve a goal “Machtavelh hholds the view thatthe prince does what he does 1 € ‘establishing and mamtaining power] n order vo gain the glory of a good reputation” (lines 37-38) Copy: © 2008 Examkrackers Inc 16. Elsewhere in The Prince, Machuvelh wrote, ‘Vereat, REASONING Waav-ur © 15 C._ the strength of his army WRONG, This was not Machiavell's goal, but the only way to acheve power Even power was not ‘Machiavellr’s goal, but the goal of the Inner Pay in /984 (lines 78-79) For Machiavelh, power was not a goal, im and of uself, but a means by which, to achieve a goal “Machiavelli holds the view that the prince does what he does [1 € establishing and. marntaming power} in order 10 gain the glory of a {00d reputation” (lines 37-38) D. his legacy CORRECT: Ths was the “ulumate goal of a ruler” For Machiavelli, power was not a goal, am and of utself, but a means by which to achieve a goal (nes 37-52) “Machtavellt holds the view that the prince does what he does [ie establishing and ‘maintaining power] in order 10 gain the glory of a goad reputation” (lines 37-38) ‘that nse quickly, just as all the other things of nature that are bom and grow rapidly, cannot have roots and ramifications; the first bad weather klls them” This most strongly challenges passage descriptions of AL the Inner Party WRONG The quote in the question does not challenge the description of the Inner Party, the government in Orwell’s 1984 No inference can be made regarding how long this government had been im power B. Agathocles CORRECT: The quote im the question does challenge the deseription of Agathocles Machravellt “gives an admurmg account of Agathocles’s nse from poverty to kingshup of Syracuse” (lines 3940) “Admiring” seems somewhat strong based upon the {quote an this question For Machiavelh, Agathocles would be doomed because he “rose quickly” CC. the totalitanan commomist government in 7984. WRONG For the purposes of this question, you should have recognized the similarity of this answer to Answer A They are one in the same. The quote 1m the question does not challenge this descnptuon No inference can be made regarding how long this government had been in power D.. Spain's easly Legalist school WRONG The quote i the question does not cchullenge this deseripuon No inferences can be ‘made regarding this school. 1ts longevity, oF ts mse to power 16 # Exancnacens 101 Passaces in MCAT Veraat REASONING 17. Acconling to the author, Machravelli would have considered all of the following to be examples of power EXCEPT A, extreme loyalty (othe ruler CORRECT: This would be an exception Mactiavelh’s idea of power. “For Machiavellian power 1s tangible and materal” (line 15) “Loyalty rs neither tangible nor materval B. strong army WRONG This would nor be an exception 1 ‘Machravelli’s idea of power “For Machiavelhians, power 1s tangible and material” (line 15) “A strong army” is tangible and material money WRONG: This would not be an exception to Machiavell’s idea of power “For Machtavelhians, power is tangible and material” (line 15) “Money” 'S tangible and material and 1s mentioned sn the passage (line 19) D. powerful weapons WRONG. This would not be an exception to Machiavelli's idea of power “For Machavellians, power 1s tangible and material” (line 15) “Powerful “weapons” are tanguble and maternal Ceppight 92008 Exarhs ace Verbal Reasoning Test 1 Time: 60 Minutes Questions 140 _ basa 18 © Bxamtcaccens 101 Passaces IN MCAT VeRBat REASONING VERBAL REASONING DIRECTIONS: There are seven passages in this Verbal Reasoning test Each passage 1s followed by several ‘questions. After reading a passage, select the one best ‘answer to each question. If you are not certain of an answer, eliminate the alternatives that you know to be incorrect and then select an answer from the remaining alternatives. Indicate your selection by blackening the corresponding oval on your answer document Passage | (Questions 1-7) 0 20 8 38 ‘There are over one hundred small white rabbits here 1m the laboratory today for the Drarze test, immobilized by their positions in theirsmall containers, with only their heads sticking out An assistant 1s placing a drop of the newest cologne or perfume directly snto each of the animal's eyes ‘The bucking and kickg of these small subjects seems to indicate that they are expeneencing severe pain as a result of tus expertment, yet at seems necessary mm order to ensure that humans do not experience eye injuries resulting from the use of this product Later, the annals will be analyzed and destroyed Ask the experimenters why they experiment on animals, and the answer 1s' “Because the animals are ike us” Ask the experimenters why s 1s morally OK to experiment on animals, and the answer 1s “Because the anunals ate not ke us” Animal experimentation rests on a logical contradiction —Professor Charles R Mage! Is tt nght under any exrcumstances to expenmment with animals? Do we have a moral obligation towards animals? ‘What is an animal” Certamly. most humans would think of these small rabbits as animals that deserve our protection But, do humans generally consider that mosquitoes, spiders, or ticks deserve the same protection? Probably not, They are not “fubsy,” the term used to describe the cuddly soft, furry, larger mammals that we generally fawn over and feel the desire to protect Recognizing this intrinsic tendency and altempung to overnde it, let us then define animals as any non-human organism Yet, this 1s such a wide definition that st could pertain to potential alters Will we witness an Alien Raghts movement soon’? We are then forced to narrow our definition of amumals to non-human orgasms that remind us of humans and, thus, provoke empathy in us However, to most advocates this Would seem rather unsatstaciory because itis not “fair” Histoncally, philosophers like Kant (and Descartes, Malebranche, and even Aquinas) did not favor the idea of ammal rights They said thet amumals are the orgame syaght & 2008 Eremkr aches, 50 6 60 65 n equivalents of machines, moved by coarse instincts, unable to expertence pain (though their behavior sometimes might deceive us into mmstakenly believing that they do) Thus, ‘any moral obligation thet we have towards animals is a derivative of a pamary obligation that we have towards our fellow humans These are the indirect moral obligations theones. Forexample, its wrong to torture smumals because it desensitzes us to human suffering and makes us more prone to using violence towards humans, Empathy as a differentiating principle 1s of little use because at 18 prumanly structural. If the ammal looks hike me, resembles me, behaves Ike me—then st must be Ike me 1n other, more profound ways However, this 1 a faulty ‘method when used to prove dentty, empathy 1s defined 1m the dicuonary as pathewte fallacy Thus way of thiaking 1s too dependent upon historical, cultural, and personal contexts. That another organisin looks like us, behaves luke us, and talks like us 1s no guarantee that it 1s like us ‘The creature 1s not capable of want, and af ut were. it would neither necessarily want nor deserve our pity. We cannot determine through empathy whether another creature, like another human, 1s expenencing pain Additionally, paun is value yudgment and the reaction to as not only relauve, but also culturally dependent. In some cases, pain can actually be perceived as postbve and be sought after If we humans cannot agree and separate the ‘obycetive from the subjective, the rational fromthe cultural — what gives us the night io decide for other organisms (without getting their approval” In Aztec culture, being chosen as a sacntice to the gods was a high honor and a burning desire We have no way of knowing. perbaps pigs prefer 10 be slaughtered We cannot decide right and wrong, good and evil for those with whom communcatton 1s barred. GO ON TO THE NEXT PAGE. 1. For which of the following of the author's assertion: NO support provided in the passage” A, Empathy 1s of hile use because wt 18 pamarily structural B. Pain can, an some cases, actually be perceived postuvely and sought after C, Human beings have an intrinsic tendeney to fawn ‘over and protect “fubsy” animals D. Another creuture is not capable of want, and af were, would neither necessanly want nor deserve ‘our pity 2. The author amples that an animal does not A. haveenough “fubsy” characterisnes tobe considered B. commumeate effectively C. benefit from human empathy 1D. empathize with humans 3. It has been said that animal experimenters “are using more and more animals whom they consider less “cute, Decause, although they know these animals suffer ust as much, they believe people won't object a» strenuously to the tozure of a pig or a rat as they will to that of a dog oF 4 rabbit” The author would probably disagree by saying that A. Dogs and rabbis ae less “eure” than pigs or tats B. People will usually object strenuously to an experiment an which any kind of ammal os soffenng C.The expenmenters cannot know how much the animals suffer 1D. The experimenters probably realize that non-hurnan organisms cannot suffer as we do 4, The passage indicates that ts author would NOT agree with which of the following statements? A, Animals communicate effectively through non: verbal means B. The reaction to pain 1s culturally dependent and relanve C. An organism may look like us, behave like us, and talk lake us, yet not be ike us at all 1D. Ananimal’s reaction to.a certain stimulus might not lead us to believe thar its expertencing pain Comnstr © 2008 Exartrackers, Ine Veeaat Reasoninc Test 1 # 19 ‘5S, The author's argument that “we cannot decide right and ‘wrong, good and evil for those with whom communication, 1s barred” (lines 70-71) depends on the acceptance of which ofthe following premises? A. The philosophers Kant, Malenbrach, and Aquinas. all urged better treatment for animal B. Rabbits undergoing Draize expenmmentation actually feel Intle or no pain yn the way humans understand aw C._Empathy is a fake method of communication D, In Aztec culture, beng chosen as a sacrifice to the ‘gods was a high honor and a buraing desire 6. The passage sugnests that human compassion for other organisms derwves from the fact that A. _helpmgother organisms may prevent us from hurting other humans, the organysin looks hike us. behaves like us, and tks ike us other organisms sutfer mn the same way bumans do other organisms usually Took like us roe 7. The author contends that in onder to judge whether we have the right to make decisions for other organisms, a researcher would need to determine ‘A. whether of not they are experiencing pain B, sf this isa denvatve of a primary obligation towards ‘other organisms whether or not we have their approval D. af empathy can be used to determine the relative fubsiness of an organism GO ON TO THE NEXT PAGE. Tests basal 20 © Exanexencnens 101 Passacts in MCAT VERBAL REASONING Passage Il (Questions 8-13) 10 1 2 2 0 38 “5 The preamble to the Declaration of Independence 1s ‘4 masterful work of Literary expression However, beyond the flowery eloquence lies great and significant meaning. No words are wasted here As it wrote the Declaration with war Ioomng on the honzoa, the Continental Congress knew America could not withstand Britain's mitary might alone. They would require allics and foreign assistance Nevertheless, they also knew America could not recewe assistance as long as the colomes were seen to be fighting a cwil War against the rung Bnsh Empire To help the colonies Would constitute interference in Great Bevtarn’s snternal affairs As Samuel Adams explained, “No foreign Power can consistently yreld Comfort wo Rebels, or enter into any kind of Treaty wath these Colonies all they declare themselves free and independent” With this 1m mind, pen ‘was put to paper When in the Course of human events, it becomes necessary for one people to dissolve the political bands which have connected them with another, and to assume among the powers ofthe earth the separate and equal station to whch the Laws of Nature and of Nature's God enutle them, @ decent respect to the opinions of mankind requires that they should declare the causes which umpel them to the separation ‘The preamble establishes the document as @ “declaration” Tt1s not some attempt to persuade and present the case for independence to a watching world, indeed to “mankind,” that the actons of these colonies are nighteous and should be supported It 1s a bold statement, with the impheation that anyone with ears to hear and eyes to see will realize that the words therein merely put to paper the “Laws of Nature and of Nature’s God” that “emule” this declaration ‘The Declaration goes so far m its assumption that all wall sec the validity i the revolution that only “a event respect (0 the opinions of mankind equates that they should declare the causes which ampel them to the separation A less bold statement, or an attempt to persuade, on the other hand, would have been a tacit admission that there ‘was another perspective on these unfolding event\—that of Bntain, which 1s never mentioned by name This was a not ‘unimportant factor to an emerging nation well aware that sts actions would be challenged and that ales would be critical to success, However, the issue, as the Declaration implies, not one of interpretation but of observation The Declaration ‘served to announce and convey only America’s side on an sssue of international law and sovereigaty, and st maintains tis perspective throughout ‘The first line, “When in the Course of human events,” elevates the Declaration to an apogee of histonal sugmificance It 1s not merely the fledgling utterances of piight © 2008 Ennmtckers, few colonies attempung to be heard, but the birthing of “one people-” Ths one people” 8 Amenca, also never mentioned by name, but bound, nevertheless, by the unification of the phrase OF no less importance, the phrase “one people” demonstrates the separateness that existed between Britain and the Americas Already separated geographically, they ‘were now two peoples who could not be made one This also served to prevent the Declaration from being, an act of cul war Finally, as Stephen E. Lucas expands upon in his essay “The Stylistic Arustzy of the Declaration of Independence.” “The most important word m the introduction 1s ‘necessary.” Revolution was not merely preferable, defensible, or 65 justifiable” According to the anternational law of nations, “necessity” was a prerequisite for taking up arms against another nation, Aga, the Congress recognized that m order for the revolution to succeed 1thad to be not only allowed by other nations, but aided The pen—not only mightier than the sword, but also a presequysite for yustifying its use. 20 8, The central thesis of the passage 1s that A. the colomes hoped to enlist Brtain’s help as an ally 1m the efforts to secede the allies. required by the colonies, must perceyve the manifest nghiness of the icependence efforts. Ca strong and unequivocal declarauon of cwil war was a necessity forthe colonies the preamble to the Declaration of Independence could not have been better written B. S Implicit in the statement that “a decent respect 10 the ‘opimons of mankind requires that they should declare the causes which impel them to separation” isthe idea that the colonies A. were required by the international laws of that time to declare why they were separaung. B. were respectful of the opinions of frend and foe alike believed their aforementioned “causes” to be self- ‘evident but were sharing them with mankind out of respect D. believed thetr aforementioned “causes” would impel them ta separation GO ON TO THE NEXT PAGE. 10, Acconiing to the passage, the proper wording of the Declaration was critical t0 the colonists’ independence cfforts because they. D. 1. would require alles and forexgn assistance could not afford to have the efforts construed as acts of ewvil war HI did not want to appear to be attempting, to justly and persuade only Tand TI only Tand 1M only 1.11, and 11) 11, What does Samuel Adams's notion that “no foreign Power ‘can consistently yield Comfort to Rebels, or enter into any kind of Treaty wath these Colomes till they declare themselves free and independent” (lines 12-15) imply about foreign interference? A B. c Db. ‘That foreign powers would not yield to rebels in their own country That the colonists considered themselves 1 be rebels who were not yet worthy of a treaty. ‘Once the colones had proven that they could win, they could then expect foreign ard, ‘That foreign nations would not help the colontes in a cwvil uprising 12, Which of the following statements 1s NOT presented as cevidence for the significance of the phrase “one people”? A “One people” served to demonstrate the separateness that existed between America and Britam at that fume ‘The phrase served to prevent the Declaration from being an act of exvil war. ‘The phrase emphasized the characteristics of cequalhty, justice, and liberty inherent an the new nation, Though never mentioned by name, America 1s bound bby the unsfication of the phrase Copynga 8 7008 Bxamirschers ine Veseat Reason Test 1 # 21 13. The passage suggests that the perspective presented throughout the Declaration A D. eloquently brings forth the stylistic artisry of the men and women who lived during this turbulent period in our nation’s history, 1s balanced rs one-sided carefully addresses the validity of the arguments of both sides. GO ON TO THE NEXT PAGE. Lassi 22 © Bxanexeacéss 101 Passaces in MCAT Vi Passage Il (Questions 14-18) ‘Arguments abound over whether manyuana should be legalized Many of these arguments pertain to the federal ‘guidelines for lengthy pnson sentences meted out for what 1s considered a relatvely petty enme. Others point out that maryvana 1s a drug that could, and should, be used for ‘medicinal purposes But most proponents of legalization gore the mounting evidence that posnts to the long-term damage to the user and fo society as a whole In the Netherlands, maryuana has been legally 10 available since 1976 “Coffee shops” sell cannabis over the counter in many parts of the country However, more people have ined cannabis since it has been legalized At the Centre for Drug Research at the University of Amsterdam, ‘Aryan Sas and Peter Cohen divided Amsterdamers surveyed 18 regarding thetr maryuana use into (wo proups—those who ‘were bom before 1958, in other words who were 18 or older in 1976, and those who were born after 1976, for whom cannabis has always been legal Only 19 percent of the Older people had tied cannabis, compared with 38 percent 20 of the younger group It would seem then that legalvzation promotes experimentation with cannabis, sf not also even harder drugs, ‘Medical research has repeatedly provided evidence that ‘maryuans use causes permanent physical. psychological, 25 and thus emotional damage to those who regularly use 1 ‘Stuches at the University of Maryland and UCLA indicated that the regular smoking of only two manyuana cigarettes a day would tend to promote toe fungus and thrush But over the years, much stronger claims have surfaced for example, 30 that heavy maryuana users perform poorly at work or school ‘and are more likely to be delinquent and develop psychiatric problems, or have abnormal brain waves Repeatedly, however, such studies have encountered the same question ure the problems caused by smoking maryuana, Or 1s 11 ust 35 that people with problems are more likely to end wp using, manyvana heavily? Manyuana 1s addicuve According to Wayne Hall, director of the National Dnug and Alcohol Research Centre at the University of New South Wales, Sydney, cannabis 18 49 not generally regarded as a drug of dependence because st ‘does not have a clearly defined withdrawal syndrome But that, he says, 36 an old-fashioned definition of addiction “While there may be debate about whether there 18 a ‘cannabrs withdrawal syndrome there 1s no doubt that some 45 users want co stop or cut down their cannabis use and find it difficult or umpossible to do so, and they continue to use ‘cannabis despite the adverse effect thas on thesr ives.” Research nto maryuana’s use as a medicine has proven inconclusive or has tended to show that ats side 50 effects rendered canals unsuitable as a medicinal drug For instance, one study surveyed the use of eannabinoxds to Copyeght © 2008 Seamkrackers, nc 2. REASONING ‘combat nausea following chemotherapy While the tablets for unjections of cannabinoids were shghtly more effective than standard treatments, their side effets, along with the ‘58 recent development of other, more powerful drugs, makes, them a poor choice for nausea relief *Cannabinoxds were ‘no more effective than codeme in controlling acute and cchronte pan and they had undesirable effects in depressing the central nervous. system” comments E1je Kalso of 60 Helsinks University Hosprial in her study Yet, under mounting pressure, the U.S Drug Enforcement Agency has reluctantly agreed to provide funds for further testing the efficacy of manyuana as a medicine. ‘One common finding of the medical research 1s that 5 the few useful components of marijuana might be extracted ‘and administered in an oral or injectable form. Given the commonly known dangers of cigarete smoking, the smoking of @ filteriess “jomnt” of maryuana seems a high: risk exeresse, However, 11s revealing fo mterview those 70 who actually want to use maryuana as medicine. With few ‘exceptions, they reyect this simple finding and mdicate that actually smoking the maryuana 1s the only acceptable form of mediemal administration ‘This brings us back full excle to the question, Or 1s st ust that people with problems are 78. more likely to end up using maryuana heavily? 14, The author claims, “Given the commonly known dangers of cigarette smoking, the smoking of a fiterless ‘jomt” of ‘maryuana seems a high-nsk exereise” (lines 66-69) The support offered for this conclusion within the passage 1s: A. weak; there 18 no evidence that smoking maryuana cigarettes 1s dangerous B. weak; the dangers of cigarette smoking are not commonly known C. strong, the dangers of cigarette smoking are ‘commonly known D. strong, the dangers of reefer madness have been well-documented, GO ON TO THE NEXT PAGE. 15, 16. What s the author implying when referring o information that people who want to use manyuana indicate that “actually smoking the maryuana 1s the only acceptable al adimunsstration” (Lines 70-73)” 18 a legitimate alternative pomt-of-view B. That people who want to use maryjuana are stupid C. That these individuals are more anterested in recreational use than in the medical benefits of rmaryuana, D. That people with medical problems are more likely to end up using manjuana heavily. Which of the following 1 offered as support for the statement "“maryusna’s use as a medicine has proven inconclusive of has tended to show that is side effects rendered cannabis unsuitable as a medical drug” (lanes 48-50) A. Cannabinoids were only slightly more effective than codeine in controlling pasn, B. The central nervous system was depressed by maryuana C. Maryuana use causes toe fungus and thrush, which in tum cause emotional damage D. The DEA has reluctantly agreed to provide funds for further testing hn 2008 Frat ackers, oe W. 18, Veteae Reasonins Test 1 # 23 Assume that the following statements are true, Which fone i$ anconsistent with the assecton that “legalization promotes experimentation with cannabis, af not also even harder drugs” A. In the Netherlands today, hard drugs are more difficult to find than in 1976 B. Inthe Netherlands, more people have tned cannabis since it was legalized C. Inthe Netherlands prior to 1976, drug use i general hhad dimumished, D. _Peoplewholivein Amsterdameome nto contact with a wider vanety of recreational drugs nowadays. Anargument FOR the legalization of martjuan, menttoned 1m the passage but not expanded upon, 1s A. the medical nses of maryuana for nausea relict B. the recent support of the DEA C. the benefits of hving in Amsterdam D. lengthy prison sentences for drug offenders, GO ON TO THE NEXT PAGE. f basa | 2A © Exanwnacens 101 PASSAGES iN MCAT Versat REASONING Passage IV (Questions 19-23) [Un 1996, New York State Govesnor George] Patakt announced a plan to reduce the cost of care in the states hospitals. The plan 1s an response to the state's perceved explosion in healthcare spending The plan's savings {come largely in the form of deregulating hoypral rates and reducing subsidies for hospral medical tanning. Under the plan, rate regulation for pavate msurance wall be phased ct and val eventually be replaced by competitive bidding Thus, msurance companies will be able to negotiate bullk- 1a rate discounts wth dividual hosprals In addition, Medicard subsidies tor medical tainmg at state hoxputals wall be reduced ‘This plan cam be expected to save some mon particularly for New York State itself. through the reduction 15 in Medicatd subsidies It wall probably save more than the current regulatory system, in which total New York State medical spending from 1980 to 1991 outpaced national spending by 22% Unfortunately. st probably won't achieve a «uly dramaue savings (aside from the Medicaid savings from 20 the subsidy cut) From 1980 ro 1991, New York's groweh in hospital costs only slightly exceeded that of most other states, ‘which operate on @ competitive system “The major concern that this plan wall shift rather th reduce medical costs, thereby creating winners and losers 25 Among the winners wall be, predictably, the state itself, ‘which will reduce the amount 1€ pays for hospital training subsidies through Medicaid (the nationwide program that subsidizes medical care for low-income residents. particularly for emergency hospital care) Insurance camers 30 ate also expected to benefit, by exploring the state's hugh concentration of hospitals, insurers will probably be able to negotiate discounts for chents” hospital care, thus cutting costs and increasing their profit margins, Those clients themselves may also share in the savings, depending on 35 what percentage of the discounts will be passed along t0 consumers On net, 115 anticipated that most wall probably enjoy somewhat lower premums Much of the “savings” will fall on the shoulders of New York Ciry, which wall have co scramble to subsidize 40 costs for the poor and uninsured, who will be turned aay from hospitals Hospitals may opt not to tum away any id, an fact, wall be prohibited from turrung away certain exuemely low ncome pauents, whose night 0 certain emergency care 1s protected by the state’s Hall-Burton law 45 Of course, should hospitals continue to offer access to the uninsured, they will effectively be subsidizing the poor and ‘will be rewarded with lower profit margins Some may be forced to close, Undoubtedly. the greatest impact wall be felt by the 50 ummsured—ofien the poor and elderly Since they do not enjoy the negotiated rates set by insurance companies, Comunght © 2008 Exambeacken, fae ‘costs to the uninsured may nse to cover hospitals" reduced profits on insured patients Simple economics dictates that uncompensated care will be reduced ay a result of lower 165 profit margins, However, the plan docs not take into account the historical fact that, of the increase in the cost of medical ccare over the past 17 years, the portion attnbutable (0 nsing salanes among doctors and other hospital staff amounts 60 10 only 18.5%, and the portion attributable to insurance costs. and ansurance abuse only 122% Over 55% of the merease 18 durecly related to the staggering cost of new riedtcal technologies The unavoidable fact remains that, 1H Americans insist on receiving state-of-the-art treatments, 685 the money for them wall have to come from somewhere 19. The passage argument suggests that which ofthe following ‘might benefit from the proposed plan? T the state of New York TH msurance carriers IL those who have insurance A. Tonly B. Land Honly C. Tand ILonly D. Lf,and Itt 20, According to the author, the governor’s plan “probably won't achieve a truly dramaue savings” (lines. 18-19) ‘The primary argument given is that: A. New York's medical spending from 1980 to 1991 ‘outpaced national spending by 22 B, though state hospital costs compose the bulk of New York's costs, these costs will be relatively unaffected. by changing to a competuuve system C. State medical costs cannot be changed through deregulating hosprtal rates D. New York's growth im hospital costs are the result of| Medicaid subsidies, GO ON TO THE NEXT PAGE. 21. According tothe passage, thedecrsivefactorm determining, whether a patient in New York City will be turned away from the hospital 1s whether this person requiring care A. has nsurance or not. B. 1s pooror elderly Chas an extremely low income D. insists on receiving state-of-the-art treatments, 22, Asvume that the telephone industry. which had been heavily regulated, was subsequently deregulated and forced to operate on a compentive system History shows that consumer (elephone rates thereafter skyrocketed and efficiency dimiatshed, never again approaching the lower rates or greater efficiency achieved dung the period of regulation ‘This finding, A. amereases the probabulity that the greatest impact willbe felt by the poor and elderly B, imereases the probability that total New York State medieal spending will probably be less thar wt 1s ‘under the current regulatory system C. decreases the probability of saving throvgh reduction in Medicaid subsidies D. reveals nothing about the probabs siate’s hospatal costs of New York Cop ght © 2008 Exaricackers, ne Verena, Reasoninc Test 1 # 25 23, Which of the following statements, sf true, would most directly challenge the assertions of the author? A. Deregulation and competition nan econorme system snvarrably lead to greater efficiency, productivity, and savings Most health consumers in New York can be expected to benefit from somewhat lower insurance premiums C. Ita possible that hospitals in New York Cy may hhave to close regardless of how well the plan works D. New medical technologies help prevent long-term Cdebilitation and subsequent intensive care of many patients. thereby proportionally reducing medical costs GO ON TO THE NEXT PAGE. Test hasan 2b © Exanenackers 101 Passaces iN MCAT VeRsaL REASONING Passage V (Questions 24-28) Among the components of the popular judicral umage, which has engendered our endunng cultural respect for sudges as lawgivers, 16 the divided nature of the sudicral mage One ancient example 18 the juxtaposed attrbutes of the Greek goddess Athena, the ideal “judien” figure of the Greek world Some of those attributes, since lost to the common low tradition, may lend new legiumacy to the current legal system if rediscovered Athena's essential nature 1s fundamentally divided, she 10 1s at once of the people and impossibly above them She 1s undemably a goddess, whe has the ear of the King of Gods and “the sealed chambes’s Keys where Zeus's thunderbolt! 1s stored” As a paragon ot both wisdom and of war, she as wiser and more crudite than any mortal Her wlentity as 15 a woman 1s surprising im a role once reserved exclusively for men, st distances her from the commoners and imparts mystique, just as her mythue virginity adds to her aloofness from the people. At the Same time, she 1s more “masculine” than any male, she 1s a perfect warrior and leader 20 Yet what 1s strkingly incongruous with the modern amage of a distant apolitical judge 1s Athena’s simultaneous erme involvement mn the community as 2 religious and judicral admumstrator ‘The common Anglo-American law Sjudieral tradition has largely abandoned Cicero's early viston of a legal scholar active in the pobtical arena and the ‘community 1n favor of a bench composed of distant scholars who are phystcally and intellectually aloof from the hugants before them While this cultivated persona has snereased the mystique and status of the lawyer-judge, st has doue so at 80 the expense of trust and popularity Furthermore, this chorce has largely preempted one possible judicial role, that of the community mediator, which 1s the yudicral tradition Athena represented Wath al, her superinuman wisdom and power, she 1s deally suited to 35 judge by fiat, yet she conscrously legisates by consensns, ‘u early Hellenic “courtroom dramas.” she 1s often prctured selecting a jury of eitvens to help her in her decision, then persuading the losing side (0 accept her decision Such mediation 1s often useful in modern neighborhood 4 disputes—such as “nuisance” claims and sntrafamilal argumenty—in wiuch the full torce of the law would be too heavy-handed Tt offers the advantages of allowing the Parties fo have their day in “court,” but is structure—having the mediator aud the parties m reaching their own agreement rather than hand down a judgment from above—encourages active participation, and hikely leaves both parties more satisfied with their outcome than would a judicral decree 4 Copynahr © 2008 Lawakicekers fe If popular acceptance 1s the foundation of the law's cendunng power, then the image of a jusuice trained 1n the 50 academy but dwelling among the people may become the judicial ymage that endures into the next age of Amenican Tegal thought 24, The discussion of Athena and the current legal system includes the assumption that: A. most of Athena's admers were surpnsed at her ender, B. present-day justices are popular and respected, €. most htigants consider moder justices to be aloof D. Athena was the ideal “judicial” figure 25, Which of the following statements, af true, would most WEAKEN the author's contention that “a distant apolitical judge” (lane 21) has less egitumacy wth the people? A. Anenemy 1s anyone who tells the truth about you B. Theonly normal people are the ones you don’t know very wel. C.To know hum 1s to love bien, D. — Crmeismmas prejudice mace plausible GO ON TO THE NEXT PAGE. 26. The author would argue that emulaung some of the 21. “juxtaposed atinbutes (Gime 4-5) of Athena 1s important to the current legal system because it Ai c D. explains why the Greeks honored their goddesses provides for greater acceptance of udicral decisions. allustrates Athena's detached sudieral renderings indicates that current judicial decisions are poorly decided The maun argument of the passage 1s that A B. oh 208 Athena was a goddess with many ad attributes the modem judictal system, though much changed, takes its roots from the Greeks Greek yustices were active inthe pol the commumty modem judges have become more ke gods with godlike atinbutes rable human ena and \Veraat Reasowine Test 1° 27 28, The author most likely believes that one of the main purposes of the judictary should be to, AL c D. render decisions that are fair while remembering that “you can't please them all” deliver chanty to the people and heavy-handed Justice to the guilty ‘encourage active participation, then rule by fat render fair and equitable decistons that leave the majonity saisfied wath the outcome GO ON TO THE NEXT PAGE. Test basa 28 © ExarrackeRs 101 Passaces in MCAT Vere. Passage VI (Questions 29-34) Immediete postwar planning following World. War TT has been portrayed by most histonans as a confusing fume, as conflicung interests within the U.S. miltary and government ined to exert their influence on future policy There seem to be a few points of agreement among modern Isoriuns to separate the confusion of interests that marked tus period Ti appears that all the partes mvolved agreed that the need to severely curtail Japan’s military and Imulitarsm was the most important promty, and they all had 10 concrete proposals ou how to achieve demuhtarization, the aims and the means were clear However, the democrat shape Japanese government and society would be made (0 lake were not as clear sm the minds of the planners, they had concrete ideas of what a democratic country should ave— 18 exwil nights, decentrahzed government, umtons, competitive markets—but there was no single depiction of ether the goal or the best method of government for banging democracy to the Japanese. Histonan Robert E. Ward maintains that even at the 20 pre-surrender stage, there was a consensus that the Japanese mperor would be retained But Ward 3s one of only a few histonans who clams this as definite; most others save the fate of the emperor to be very uncertann, atleast until a much later date Even by Ward’s own account, Americans at every 25 level, from policymakers to homemakers, saw the emperor as the heart of the Japanese poliucal system, and the reason for al ss faults Ths assessment of the emperor as the agent of imitans, mtuonalism, and the war seems to be a strange 30 one for American political leaders to have made, 1t does not conform to the views of modern histonans about the politcal role ofthe emperor The prevailing opinion now s that Emperor Hhrobio was merely 2 figurehead forthe geno [capsalist ete}, whom he supported, and later beeame a tool £35 forthe multary and pro-mlitary bureaucracy. wth whom he disagreed bat was unvalling © formally oppose. Hision: Mikio Hane says thatthe emperor was, 1n fat, so passive that the army actwely tned to change fs image into that of a war hawk with dramate photo spreads, but that these 40 attempts were unsuccessful, and the emperor retained the repatation wathin Japan of being a stnetly ewilan figure—a dramatic dissunslanty fom his mage outside of Fapan How did this flawed perception of the emperor as an aggrandvang miltary dictator tke hold im America? One 45 possible explanation is that, because Japanese soldiers swore allegiance 10 the emperor and carried oa the war effort in his name, Amencans beheved that he was their actual leader ‘Whatever the reason, the debate over the emperor's role 1n the new Japan raged on among the Americans The question 50 was argued along a conerete line: no one actually laked the institution of the emperor, and everyone blamed him for the ‘wat, but he was seen as a crucial tool for the America Copyngir @ 2008 Sea bers, Ine to promote cooperation and bring about a democracy If the emperor could be co-opted, he could also be used as 55 a disguised clarion for American mandates to be broadcast to the Japanese, with almost divine authoruy. IF he were deposed or tred as a war erimunal, Amenea would incur the undying anger of the Japanese people, and all future reforms would have to be forced on them 29, Before the onset of World War Tl, the American public percewved the emperor as = gentle and ineffectual figurehead ‘The most reasonable explanation on the basis of passage information is that. A. the war had not yet begun B. there were no Japanese soldiers killing American soldiers in the name of the emperor CC. Amencans had always had a flawed perception of the emperor. 1D. this period, as portrayed by most histonans, was a “confusing ume" 30, ‘The author’s apparent pomtin referring to historian Robert E Ward's analysis that “there was a consensus that the emperor would be retained” (lines 20-21), 1s that: ‘A. there was consensus that the emperor would be retained, B. this analysis was of critical importance to the passage. C. there was actually no clear consensus that the emperor would be retained D. a flawed perception of the emperor by the American people GO ON TO THE NEXT PAGE. 31. IF the passage information 18 correct, what inference 1s justified by the lact that there 1s sill a Japanese emperor, ‘almost sixty years after the pertod described? A. The American debate regarding the role of the ‘emperor had been decided B, The Japanese mufitary had retained much of there political influence C. The Amencans had decided that the emperor could be used to their advantage D. The Japanese genro had lost much of thetr power 32, Acconting tothe passage, all American partes agreed that {he most important priority Was ‘A. reaching a consensus on what should be done about the emperor B. bringing democracy to Fapan C. not something that could be determined easily D. _resttrcung Japan's anlitary capuiuhiies lopgnignt 8 20 Eaumisaeeoe 2B. 3 Versat Reasons Test 1 # 29 Given the passage implication that the goal of bringing, a democracy to Fapan was a foregone conclusion (lines 11-18), which of the following passage assertions seems contrary to tins dea” A. Japan would be demubtarved as soon as possible B. The emperor would be retemed C. The emperor would be deposed D. The emperor was, an fact, passive ‘The passage suggests that sts author would probably dvsagree with which of the following statements? As ‘The emperor was unwilling to formally oppose the imulitary ‘The emperor was actually passive ‘The emperor was a member of the genro, whom be supported D. The emperor was seen by Amencans as a mulitary eictator GO ON TO THE NEXT PAGE. vest { vasa | 30 * Exanicnackens 101 Passaces In MCAT Verna: Reasons Passage VIl (Questions 35-40) 6 20 2s Fa 35 40 4s ‘The first psychologist to study memory expertmentally 1m a clinrcal setting was Dr Hermann Ebbinghaus His methodology was to have test subjects memorize word Iists Of variable length, under varying test conditions. Using this ‘method, Ebbinghaus determined that the average persons _memory span is seven unuts of information Ebbinghaus also noted that the memontzation of a series of ideas that 1s to be reproduced 1 more difficult, the longer the series 1S That 1, the memonzation not oaly requires more time (aken by sself, because each repetition lasts longer, but it also requires more ‘ume relanvely because an ancteased number of repetitions becomes necessary Six verses of a poem require for learning not only three times as much time as two but considerably more ” Invually, this would suggest that word lists are best learned in sub-lists of seven However, the increase in retention may be less dramatte. in fact Ebbinghaus ‘on to conclude that. if subjects were to memorize a words in a certain order and come back to relearn them a day later, but then were given a list of the same words out Of sequence, their rate of learning for the new list would be no better for having studied the other sequence before, the savings in time disappears completely Thus, to memonze single words, t would be more efficient to learn them one ata ume, repeating « word, say. 7 umes before gomng on the next. rather than going through the sub-list 7 times before tackling the next sub-hst Yet 1t seems that sometimes, the order in which the words are learned can dictate the ease with which they be recalled, due to the “sertal position effect" According to Columbia Universty’s Professor Herbert STerrace, when we study a string of words, we tend to remember the words closer to the beginning and end of the hist (also known as the "prumacy” and “recency” effects), but have a harder tome with those in the middle If this is true, 1t would make sense to go through a list of words, learming them one at a me, but then rearrange the list so that the elements that were in the center are now near the beginning or end. and then study the hst that way the next ume This research may validate the populanty of randomly shulfied flash cards as a study aud. ‘This serial postion theory could also explain why people advise us to spread our studying over a few days, instead of cramming a semester’s worth of information 1nt0 ‘our heads the night before a test For instance. if we try to read 250 pages of St Augustine m one sitting, we migh remember the first 25 pages and the last 20 fusrly well, but that leaves a vast 205 pages that fall into the vaguely remembered middle of the string. If we could read this CConynigit © 2008 Cvarrackers. material over a period of five days, there would be many ‘more beginnings and endings, with fewer pages falling 1m the middle. Ironically, Dr Terrace cites a “precipitous ‘58 declme” mn recall un the frst hour after learning, so students ‘would be well advised to avoid being tested until at least ‘one hour later In his writings on improving memory, Allen Baddeley stresses that monotonous rote learning, without active 60 concentration on the material, makes for very inefficient recall when compared to an interested or eritical reading. He gives the example of the mnemomist Shereshevsku, who could seemingly remember any datum for any length of ume because of his innate abulty to visualize amages relating to 6 the subject matter Baddeley write “In general, this meant that even the dnest and most unpromising material ereated a vivid experience which was represented not only visually ‘but in terms of sound and touch and smell.” 70 Baddeley counsels us to attempt to do the same, to visualize an image or story to go with a word, to stimulate ‘ur attention and bresk the monotony of a long list of data ‘This isim ine with Ebbinghaus, who noted that memorizing 2 portion of Don Juan took 10 ume’ less tume than memortzing 75. series of nonsensical syllables of simular length 38. According to the passage information, what would most likely happen if student was tested "inthe first hour after learning” (lines 54-55)? ‘A. Overall, recall would be very poor due to a decline sm this first hour. Overall, recall would be strong based on the strengths ‘of short-term memory. ‘The student would be likely to recall only the last part of what he had been studying. D. The “recency” of the testing would result mn better recall than testing done later. GO ON TO THE NEXT PAGE. 3. The assertion that Shereshevsk “could seemingly remember any datum for any length of me because of his inate ability to visualize images relating to the subject matter” (ines 63-65) 15 NOT clearly consistent with the information about, A. Terrace’s “serial position effect.” B. memorizing a portion of Don Juan CC. monotonous rote learning 1D. Badéeley’s quoted description of this abilty 37. According to the anformation about “serial position effect (live 32), readers of this passage would find 1 easiest to recall the mformation about A. Ebbinghaus’s being the first (0 study memory experimentally and "serial position effect 1B. “serial position effect” and the memory abihtes of Shereshevskat C. the spelling of Shereshevskn’s name, and that Ebbinghaus was the first to study memory experimentally D. visualyzing an image to go wah a word, and that the average person’s memory span 1 seven units of formation 2. According to the passage, prior to the memory studies of Dr Hermann Ebbinghaus, the study of memory was most probably characterized by which of the following” ‘A. Speculation based upon hitle experimentation within achinal setting. Facts observable through experimentation outside of a chmical serang. C.Bvidence based upon some experimentation wathin achimecal setting, D. Observation without expeninx chnveal setung. ration, outside of a omyng 5 ZOE Doambrackers, Ine \Vergat REAGONING Test 1 # 31 39, Based upon the Ebbinghaus quotation (lines 8-16), we ccan conclude thatthe professor equated A. decreasing difficulty with the easing speed of recall B. decreasing difficulty with the anereasing ume it takes to recall C.mereasing difficulty wath the mereasing speed of recall D. increasing difficulty wh the decreasing ume at takes to recall, 40, Passage formation indicates that Ebbinghaus"s conclusions regarding memoriang a list of words in a certain order and then changing the sequence of this hst (ines 18-29) were most hkely hased upow Lam experiment conducted 1n the manner deseribed HL, conclusions based upon his other experiments HL speculation watl no expennmentat only Monly Wonly Vand Il only pore STOP IF YOU FINISH BEFORE TIME IS CALLED, CHECK YOUR WORK. YOU MAY GO BACK TO ANY QUESTION IN THIS TEST. STOR “Test ‘assy 32 © Exancencress 101 Passacts IN MCAT VerBaL REASONING Soaynght = 7008 Fxetkeacrers 30 Verbal Reasoning Test 2 Time: 60 Minutes Questions 1-40 B zasat 34 # Exancraciens 101 Passases i MCAT VERBAL REASONING VERBAL REASONING DIRECTIONS: There are seven passages in this Verbal Reasoning test Each passage 1s followed by several questions After reading a passage, select the one best answer to each question If you are not certain of an answer, eliminate the alternatives that you know to be incorrect and then select an answer from the remaming alternatives Indicate your selection by blackening the corresponding oval ‘on your answer document. Passage ! (Questions 1-5) ‘The concepts of right and wrong and good and evil, in universal sense, are rational, and, as such, are valueless and even damaging to society. Logical concepts are based upon a series of verifiable premises that, when placed side 5 by side, create an argument leading, via the rules of logte. to a conclusion. In order for a concept to have logical clarity, the accuracy of the premises of the validity of the argument ‘may be disputed, but these essentials of logieal structure ‘must be present Traditions] moral concepts fanl am this 10 regard Universal morality 1s not grounded in verifiable facts, ‘but mn fath Its apologrsts clasm factual basis as evidenced bby overwhelming mutual agreement “We all sense morality, so it must exist” But can mutual sensation evidence fact? 15 Descartes correctly argued, “L thunk, therefore Tam "Though Descartes’s example may demonstrate that sensation can ‘evidence fact, tis the certainty Founded n self-sensation that ‘compels his argument ‘This certarmy 4s lost an the mutuality of the group. For instance, 7 ve think, therefore we are, 20 rehes upon faith inthe existence of others How can one be Certain that the thought of others snot a self: manufactured allusion? It 1s this uncertainty of mutushty that undermines any logical vahdity of a concept of universal morality Right and wrong 1n the moral sense rely upon feckugs of others 25 that can never be ascertained with confidence ‘The damage to 2 society that blindly accepts such fallacies of reason fies un the fallacy stself, sf an action 1s not truly wrone, 1 should not be formddn, punished, or otherwise condemned Doig so risks condemning night 30 rewarding wrong, and, thus, perpetuating evil Although the concepts of night and wrong cannot stand ‘alone as universal truths, right and wrong can be qualified ana deterministic sense, not only allowing moral judgments but requinng them If man 1s a produet of his biological 3 makeup as science tells us he 35, man must act to reproduce, and not just 10 reproduce, but also to create an environment that is most conducive tothe reproductive efforts of his own offspring According to the Factual findings of science, any Copy ght © 2004 Ezines 8 50 action not conducive to these goals cannot be perpetuated and 1s, 1m fact, a waste of energy that would in effect reduce the chance of aciuevernent of these goals The rules of Danwimsm dictate that animals exhibstug such behavior ‘would be less fit Such animals, thear offspring, and their behavior would be doomed 10 extinction Actions that are not couduewe to the goals of reproduction as prescribed by Darwinism are ‘wrong’ ot “evil” actions, whule those that are conducwe are ‘right’ of “good actions Upon first reflection, one might conclude that such a philosophy would prescribe a prmuve, anarchical society. However, it 1s the nature of man to be social and to work together It1s often im the best interest of the mdhvidual and his own offspring to obey societal rules and regulations even ‘when ins fitness may be reduced in the short term, Because offspring are only partially related to their parents, there ‘may be cases where an individual 1s better served to provide for his brother’s offspring or even his neighbor's offspring over one of his own resulting mu what ouowardly appears as altruistic behavior The resulting society cannot be anything bbut what we have today The very existence of our society 15, mn itself, the proof = Which of the following assertions in the passage 1s NOT supported by an example, argumeut, or by reference to an authority? A. The concepts of night and wrong are tradoual moral concepts B. Irrational concepts are damaging 10 society C. Right and wrong im the moral sense rely upon feclings of others that can never be ascertained wth confidence. D. _ADarwinsan philosophy would prescribeapamuve, anarchucal society GO ON TO THE NEXT PAGE. 2. Which of the following would the author believe 1s the most important factor © eonsider when making a decision concerning a moral dilemma? ‘What others in your position would do What makes you feel good What 1s best for society What 1s best for you pOR> 3. A rehgious man says, “It's wrong to have sex betore ‘marage." The author would most hkely argue. A. Sex before mamage rsalways nght because 1 results| an reproduction Sex before mamage may or may not be wrong, depending upon the circumstances ‘There 1s no right or wrong concerning sex before D. Sex before mamage 1s only right af a child 1s the result ynght © 7008 Exarhracker, Versat Reasonines Test 2 # 35 4. Based on the passage, which statement could most reasonably be attributed to the author? A. People who produce offspring are naturally good B. People who produce offspring are naturally bad C. People who produce more offspring are nesther bad nor good; they just are D, Modern socices are morally better than ancient Which of the following 1s the most logical conclusion when applying the author's definition of morality (ines 45-48) to animals? Only extinct ammaly were moral If anumals are prohferating, then they are amoral Anumals cannot be moral Tfammals ace proliferating, then they are moral GO ON TO THE NEXT PAGE. { cisu, 36 © Exannnacens 101 PASSAGES i MCAT VeRtAL REASONING Passage Il (Questions 6-12) 10 5 2 38 4 50 CCopynaht $ 2008 Exorkvackers, Indusrial melanism 1s a phrase used to deseribe the evolutionary process whereby iniually light-colored organisms become dark, as a result of natural selection, 1m Dhabutats that have been darkened by soot and other forms of dustrial polluwon ‘The significance of mdustrial melanism 1m the European peppered moth, Biston betularta, as one of the first, and still most exted examples of “evolution 1n action,” places importance on the need to be sure that the Mory 1 right In the $0 years \ince Bernard Kettlewell’s pioneering work, many evolutionary biologists, particularly in Briain but also in other pans of Europe, the United States, and Japan, have studied melanism an this spectes: Ther findings show thatthe precise description of the basic peppered moth story 1s wrong, maccurate, or incomplete vwith respect to most of the onginal story's component pars The story actually began 1n 1896, when J W Tutt noted thatthe then-typical. light-colored European peppered moths were well camouflaged against the light-colored foliose Iichens that grew on ee trunks im unpolluted woodlands. But in woodlands where dustrial pollution had kalled the lichens, exposing the bark and darkening the ree trunks, the datker-colored _melamcs were better camouflaged, and, he felt, more prevalent Since conspicuous moths are more likely to be eaten by predatory birds, Tutt attributed the merease in the proportion of melame forms to natural Selection In the 1950s, ingued by ins own observations of this phenomenon, Kettlewell tested the idea experimentally by marking several hundred peppered moths (typicals as well as melanies) and releasing them onto tree tronks 1m a polluted woodland near Birmmmgham, England Ketlewell observed through binoculars that metamies seemed less conspicuous than typicals and that birds took conspicuous moths more readily than inconspicuous ones. That night he recapiured27 5 percent of the melames, butonly 13 Opercent of the typrcals. suggesting that « much higher proportion of melames had survived predation Ketilewell later repeated this experiment in uapolluted woodland in Dorset, England, ‘where the recapture percentages were the opposite of those obtamed in Birmingham He concluded that “birds act as selective agents. as postulated by evolutionary theory.” and that industrial melanism was “the most sinking evolutionary change ever uctuslly witnessed m any organisa” However, when biologists looked beyond Birmingham and Dorset, where Kettlewell had conducted hisexpersments, they found discrepancies im the expected geographical distnbution of melanie moths In rural East Anghia, where there was hitle mdustnal pollution and the light-colored typical seemed better camouflaged, dark-colored melanie actually reached a frequency of 80 percent. This led DR. Lees and ER. Creed 10 conclude that “erther the predation experiments and tests of conspreuousness to humans are misleading, or some factor or factors in addition to selective 55 i * predauion are responsible for maintaining the high melanie frequencies” By virtue of the fact that Kettlewell performed his ‘own observations via binoculars using hus own natural eyesight, one can surmise that he assumed that a bird's visual perception and abilities were the same as a human's, ‘and further, his own But, humans and birds see the resting places of moths differently in ultraviolet (UV) light For birds, bluck pepper moths are actually camouflaged im unpolluted forests on foliose in pure UV, but the reverse 1s true in human vision In his expermments, Kettlewell released moths directly ‘onto tree trunks, and acknowledged that they “were not free totake up theirown chorce of resting site Cadmit that, under their own choice, many would have taken up postion higher inthe trees” Before the 1980s, many investigators continued to find st conventent 10 conduct predation experiments using dead specimens glued or pinned to tree trunks This araficial placement 1s crtically misleading for to reasons First, as explained previously, an incorrect assumption 1s smmediately made that bueds perceive the moths 1n the same way that humans do Secondly, one naturally assumes thar ths as where the moths will actually be found Yet. 1m 25 years of fieldwork, one promunent researcher and his colleagues found only one peppered moth on a tree trunk, ‘The moths rarely choose to alight on vertical surfaces, but anstead rest high in the understory of tees on the bottoms of the lateral branches. Significantly, the foliose lichens are nonexistent there 6. According to the passage, which of the following 1s most Likely to be true about the relationship between the foliose lichens and the peppered moths? A. Typrcals are more difficult for birds to see on unpoliuted lichens Moths and hechens t areas C.Lichens in polluted forests provide concealment for rmelamies, 1D. Thew relationship 1s based upon Keitlewell’s supposition \d to thnvean the more polluted GO ON TO THE NEXT PAGE. 7. The author 1s prmanly concemed with demonstraung ‘hat ‘A. Poor experimentation leads to poor findings B. Industry fs had litle effect on populations C._Kettlewell knew that his expersment was maccurate ‘and that his findings were wrong. D, The scientific communsty can be fooled. 8, The passage suggests that the discrepancies nthe expected geographical distribution of melatuc moths can actually be attributed «© A. the relanve nonexistence of foliose lichens 1n the derstory differences between the way humans and birds see, which Kettlewell was not aware of C. selective predation D.__ This information 1s not provided. 9. According to the author, melamies and typicals tend to rest on vertical surfaces. feed on foliose lichens rest where lichens are nonexistent thrive mn ndustnally polluted areas eoee 10, The author suggests that Kettlewell knew that certain aspects of ns experiment were smaccurate B. did not know that aspects of Ins experiment were snaccurate GC. did not think that his findings were very sigavficant D. was devious, Cepynght © 2008 Exam beaches Ine Versa Reasoning Test 2 * 37 11. According to the author, why 16 important to determine Af Kettlewell’s story s right” It Kettlewell’s story 1s flawed, then we must accept that evolution 1s awed Industnal melanism 15 significant ‘The European peppered moth story 1s sill one of the most cited examples of “evolution m action.” Most biologists sight Furopean peppered moths ‘when they are working on stores about industnal melanism 12. Asitisused.in the passage, the phrase mdustrial melanism refers 10 A. B, © D. the relationshup between the foliose and the moths hhabntats that have been darkened by soot and other forms of mdustral pollution a process of natural selection, whereby habitats that have been darkened by soot and other forms of ndustnal pollution attract dark-colored organisms an evolutionary provess GO ON TO THE NEXT PAGE. Test 2 38 © Examwnacens 101 Passages iN MCAT Verba REASONING Passage Ill (Questions 13-18) 10 18 % % 40 45 50 Thove mvolved in the producuon of law and legal texts made 1t a point to set the language of the law apart from all other types of writing There has been a ‘continuous philosophical determination, both conscious and subconscious, to present the law as something distinct, from “ordinary” language, and thus above the reach of the “Tait” ‘The pronouncement of ancient law was often attributed (0 2 divine lawguver, or else a messenger with ‘2 sible connection to the divine or supernatural Their commandments and prohibitions were transformed into @ binding “aw” by an external authority, “the lawgrven” of, ‘more preeisely.the ancient community's shared belief in the Jawgiver’sintrnsic power, omniscience.or justice Examples of the supernatural-authortative lawgiver abound the Bible (direct word of God), Moses (messenger of God), Christ (on of God, miracle worker), Athena (goddess, masculine woman, supernatural birth, messenger of Zeus), and the seer ‘Twrestas (venerable, blind, visionary, hermaphrodite) are Just a few examples Someumes, of course, the lawgwvers were undeniably human figures. such as Roman Emperor Jusuman, the Enghsh kings, or even the town “elders” Yet even then, devices were constructed for them 10 forge a public link with the divine Roman emperors typically acted as High Ponteff m taking auguries, while rulers had prophets, prests, and the “divine right of kings.” and even ‘old men had “benches of polished stone n the sacred circle” fon which to sitin borrowed glory Thus, ancient law, with 1's fundamental reliance on external authonty, had litle need to ustify ts content snternally Since they drew their legitimacy from fiat —that 1s, their lack of internal reasoning oF evidentiary support— ancient laws tended to be simple and absolute They were also comect, by defimuon A quintessential example of ancient law is the single-minded rule ofthe demomic Furies “every mortal soul/ whose pride has once transgressed the law of reverence due/ to parent, god, or guest shall pay Sun's just, mexorable toll” Thus, under ancient law, the innocent are enuely vindicated (“The man whose open handy ate pure/ Anger of ours shall not pursue.” say the Furies), while the guilty are usually killed outright Because thetrs 18 & tradition of focusing on absolutes, the Funes are poor at differentiating degrees of guilt and equally poor at articulating the reasoning for their justice It was the English Judge Thatcher who heralded an ‘important turning point in legal reasoning by wentfying himself as an independent legal authonty, by virtue of his own self-created legal abulity Yet 1t was Teft to the later Judge Edward Coke to define the mode of reasoning that ‘would dominate the common law for mote than three centuries As he prepared to rule ona legal case, King James | was doubtless surprised by Judge Edward Coke’s sudden Copyright ©2008 Exarivackers, Ire ‘pronouncement that he could no Longer do so, because Coke hhad perfected a kind of logic, “aruficial reason,” which ‘55 was So unique as to make him and hus fellow judges the sole proper interpreters of the law. The exact nature of this new method is rot ammediately ascertainable from Coke's desertion reason, which 1 to be understood as an 60 aruficial perfection of reason . gotten by Iong study, observation, and experience and not every ‘man’s natural reason, for [no one 1s born skatiful) ‘Thus legal reason [is the highest reason].” Yet what 1s discermble 1s that this method 1s 50 65 esotene and exclusive that only professional lawyers can comprehend and use it. It was for later legalsts to actually invent its processes, but for now, Coke had elarmed for the legal profession an exclusive monopoly over i. 13, The central thesis of the passage is thet. A. recently, there has been movement towards rendering legal language more accessible to the “Iaity” B. lawgivers and judges have usually considered themselves to have a divane night C. _since the earliest umes, the language of the law has needed to be practical and precise D. throughout history, the language of the law has been ‘made purposely beyond the understanding of those Jacking 2 legal background GO ON TO THE NEXT PAGE. 14, Which of the followsng clarms is NOT expheitly presented 1m the passage as an example of an “undeniably” figure? 1 Roman Emperor Justiaran TL Enghsh kings LIL Moses. Lonly Tonly Monly and Tl only 1S. According to the passage, which of the followmg Statements would be the most accurate m desenbing “the ‘single-minded rule of the demonte Furies” (line 35)? A B. The accused was either completely guilty or completely snnocent ‘The guilty stood to benefit because the Funes sought a punishment which fit the erie. The innocent were often punished because of the Funes disdain for mortals ‘The Furtes would expound at length upon the reasoning behind therr decisions 16, Which of the following statements 1s the most reasonable that can be drawn froan the author's description of King James T's surpnse at Judge Edward Coke's pronouncement (Lines 5 6)? From that point forward, King James was the sole snterpreter of the law, From this point forward, King James 1 demanded nclusion in the judges’ mnterpretation of the law, Prior to this, King James’ Tyudges had not been the ‘only interpreters of the la Prior to this, King James I had allowed his judges to be the sole interpreters of the law Veneat Reasoning Test 2 * 39 17. Though not mentioned in the passage, the biblical Ten Commandments can best provide an -cample for whi ‘one of the following passage statements? AL B. 18. The author's assertion that “Someumes, of course, the undemisbly human figures “The pronouncement of ancient law was often atirbuted to a divine lawgiver, or else a messenger with a visible connection to the dine or supernatural” “There has been a continuous philosophical determination, both conscious and subconscious, to present the law as something distinct from “ordinary” language” “Thus, under ancient law, the innocent are entirely vindicated lawawers were sven old men had “benches, ‘of polished stone in the sacred circle” on whch to sit in borrowed glory” (lines 26-28), supports which of the following conclustons? A These elders required a bench im a sacred place befitting their divine authority ‘The benches alone would have imparted a divine connection to these human figures ‘The circle “became” sacred in the presence of these divine elders. ‘These undemably human figures gleaned external authonty Irom a divine Focation GO ON TO THE NEXT PAGE. asa 40 © Exarnneaceens 101 Passaces int MCAT Versa RERSONING Passage IV (Questions 19-23) In society, people tend to base their decisions not on what trie, but rather on what they percewe to be true. Sometumes. the two are the same, making the decisions comect But often they are not Thus, many decisions are 5 based on first impressions. winch are themselves predhcated ‘on mcomplete information, stereotypes, or even wild guesses extrapolated from a few facts With this realzation, the emerging science of impression management 15 concerned vith teaching people t0 act in ways that trigger positive 10 responses from others and to avord common negauve stereotypes assoctated with certain acts or features The most important smpression 1s the first one First nptessions (end to be espectally lastung, both because people tend to trust their frst instincts as correct and 15 because once a person forms a definite positive or negative Impression, he subconsciously seeks to validate it by perceiving furure actions in a way that 38 consistent wit that umpression For example, once an snterviewer decides fan applicant 1s competent, he will view the appheant’s 20 later questions as demonstrating valid curiosity. but af the interviewee concluded the interviewer 1s incompetent, he ‘will perceive the same question as demonstrating ignorance of baste facts Impression management 1% the stockanstrade of 25 courtroom consultants. since the courtroom 1Sa place where yurors make major decisions about hisgants* eredibilty, demeanor, and worth, often based on nothing more than a few days’ rehearsed testimony One of the proneening writers ‘on impression management 3s the jury consultant Jo-Ellen 30 Dimitrus According to surveys conducted by Dimatrius, the ‘main qualities each person should try to proyect are. m order of importance. (1) trustworthiness, (2) compassion, and (3) humility and/or competence Trustworthiness consists Cf both honesty and rehabihty (most visibly demonstrated 35 by keeping promises) Compassion 1s marked by canng, kindness, and gractousness, Competence tends 10 be inferred from some mx of visible intelligence, confidence, and demonstrations of actual competence at a task People should at all umes strve to project trust and compassion, 40 since the lack of these qualiwes 1s always evaluated negatively However, since visible humuluy may suggest less competence, people must sinke @ balance between these two. based on Whichever trait 1 more important t0 the observer's needs. Generally, sn informal soctal context. 45 people should display humulty rather than capability, since ‘Social acquaintances are relatively unaffected by a strangers competence but will judge jum arrogant for blatant self promotuon Professionally. people may choose to emphasize eather their competence or their carny/humlity, depending 50 on which (rat people value more in their profession For ‘example, a personal injury htugator may want to project 2 ‘ough and competent image to suggest he sa hard bargainer Dikely to negouate a large one-time settlement, whereas 2 Copyright © 2008 Bramitackers re rurse of psychotherapist will want to strive for a “nicet 55 image, even at the risk of seeming “softer” ‘There are a few behaviors that seer to always improve ‘one’s impression, regardless of circumstances These include direct eye contact (which 1s invanably perceived as more honest), smiling, good posture, shaking hands, and 60 appropnate enthusiasm. Similarly, there are “toxic traits” that, with the same invariabilny, detract from one’s image ‘Among these are bad grammar, cursing, sarcasm, obvious aggression, and visible anger. Nonetheless, most traits and behaviors tend to convey some good qualities, but also 165 unavoidably suggest some undesirable connotations. This 1s due largely to the Fact that traits suggesting competence and capability tend (© conflict with the aspirational goal ‘of humility For anstance, a man whose image 1s wealthy, sophisticated, and dominant conveys power, confidence, and 70 leadership, but may seem domineering, arrogant, and aloof, especially to observers that are more low-key It he changed his image by wearing more casual clothes and adopting a ‘quieter, mote subdued approach, he would gain the latter set of desirable associations, but lose the former The key 75 10 effective impression management, therefore, 1s to find an equilibrium between these traits by assessing which 1s ‘more likely to be valued by the intended audience under the ‘circumstances, and emphasizing those qualines 19. ‘The ideas of the author m the passage seem t0 derive prunarly from ‘A. evidence on how people aet B. speculation based on observation C. surveys conducted by a researcher D. _ facts observable inthe courtroom GO ON TO THE NEXT PAGE. 20. ‘The central thess of the passage 1s that 1 aperson can learn to ‘read’ others order to gain a more accurate impression of them IL a person can learn to convey the appropriate impression to others TM a person can Tearn to reall” the ampression he as conveying to others. only only Wonly Tand tT only 2A The passage discussion most clearly suggests the hypothests that first ampressions are A. B. c Dd. trusted by those who make them and usually negative Jong lasting and always harmful ‘often wrong. Jong lasting, and difficult to overcome ‘often wrong, trusted by those who make them. and. always harmful Versa Reason Test 2 * 41 22, What disncon 1s amplied m the passage between professional and informal contexts, respectively” Sor Enther competence or humility and self-promotion Always competence and hurmlity Canng/humsty and humility Deciding on which trait 1s valued more and humility 23, According to the passage desenptions, which of the following “main qualines” (lane 31) would be the most ulficult to convey ma shor first ampression? A. B. c D. Trustworthiness Competence ‘Compassion Humlity GO ON TO THE NEXT PAGE. yest2 asa 42 © Exanexnacnens 101 Passaces in MCAT VeRpat REASONING Passage V (Questions 24-29) 10 5 Fy 30 38 «0 “6 50 Cooynght @ 20 Using extensive survey data, c Dr Linda McCallister has identified several disunct “styles of spoken communications and has studied their interaction Her book, Sas What You Mean, Get What You Want, offers a guide to understanding colleagues with different speah siyles and avording misunderstandings based on stylistic chfferences She identifies three basse speaking styles, Any person 1 capable of uulzing all three and may use any one ‘oceastonally, but everyone has one preferred, or “dominant.” style The so-called “Noble” style 1s typically employed by those who believe the m ‘of communication 1s, to speak the truth and achieve results quickly This style 18 characterized by blunt but honest content, spoken briefly and wuthout great detail An example of the Noble style any role played by tough cowboy Clint Eastwood, who wastes no ume telling people hus honest opinion of them Its generally the style prevalent among males Another baste style, favored by those who believe the primary aim of talking 1s © maintain friendly interpersonal relauonships, 18 the so-called “Reflective.” identifiable by pleasant, polite, and diplomate speech, along with a reluctance to tell people anything that might upset them This style 1s best ulustrated by soft-spoken comedian Woody Allen but 15 the more dominant style among women. The third basic style 18 the “Socratic.” chosen by those who believe that conversation, debate, and detailed understanding should be nofcommunication Socrates are much more verbose ‘and rambling and tend to focus on detail and anecdotes. as well as rhetoncal questions and linguistically sophisticated arguments Professors often adopt a Socraie role. encouraging debate to share ideas and sharpen rhetorical skill. In addition to the pure styles. there are two “mixed” or blended styles The “Magistrate” combines the Nobles candor with the Soeranc’s verbosity, creating an honest but long and often repentive speech pattern calculated to win” conversations by perwading others to adopt the Magistrate's own (honest) opinion A prime example would bbe argumentanve talk show host Laery King The other syle 1s the “Candidate,” who blends the Reflective's miceness ‘with typically Socratic chattiness, and who thes to persuade others to adopt hrs posittons by first charmung them with bis politeness and humility the a According toDr McCallister, no onestyles necessarily “correct” or superior. each one’s view of the proper role fof communication may be valid, at feast under certain curcumsiances However, the difliculty of communication 1s that people of each style tend to preter speaking 10, and listening to, their own style only Frequently. when confronted with a different style they react with annoyance and unwarranted assumptions, not realzzing the validity and intentions of other styles For example, the diplomatic and patient Reflecuve often misperceives the blunt an Examkrackers, Ine concise Noble as tactless and abrupt, whereas the Noble 155 may merely be honest and efficient. Simularly, the bottom— Iune-ontented Noble 1s frequently annoyed at the Socratc’s verbosity and fixation on details, not recognizing it as an attempt at thoroughness. Dr, McCallister’ research carries several implications First, by recognizing each style’s ‘60 inherent strengths and weaknesses, students can tailor their manner of speaking to meet listeners’ expectations For ‘example, if'a person works 1n a field where “toughne expected and lauded, he should probably conform to that expectation by favoring the Noble style The same person {65 might wish to switeh to the more pleasant Reflective style socially, or when applying for a position which requires compassion. Moreover, communication 1 enhanced, and misunderstanding and fretion reduced, when speakers meet each other's expectations by mirroring each other’s speaking 70 styles Since a speaker cannot practically expect to change the other person's style n the course of a conversation, he should adapt his own to achieve rapport Sumularly, students exposed (0 the multipleity of styles, and taught the valid reasoning behind each, are more likely fo develop tolerance 75 for styles other than their own, 24, In the context of the passage, the word fionest (line 14) means A. not telling les B. caring enough to say what you mean C. telling the cath D. saying what ‘consequences. you mean regardless of the GO ON TO THE NEXT PAGE. 8. 26. Which of the following statements most strongly challenges one of the assertions made in the passage” A. A Socratic will tend to engage in a controlled, concise debate B. Clint Eastwood's natural style 1s Socrane C. Bach of the spoken styles has s place 1D, Reflectves are patient and diplomauc According to passage descriptions, the tile of Dr MeCallister’s book Say Whar You Mean, Ger Whar You Want would most characterzze which of the following styles?” AL Candidate B. Reflective C. Magistrate D. Noble In order to Get What You Want, which of the following. styles would probably be the /easr effective? AL Candidate B. Magistrate ©. Socratic D. Reflecuve ight 27008 Een Verpat REASONING Test 2 * 43 28. According to the passage, “Frequently, when confronted with a different style, [people] react with annoyance and unwarranted assumpuons, not sealing the validity and intentions of other styles” (lines 49-52) ‘The ‘reaction’ of a male Reflecuve would most Iikely manufest itself through A. keeping an opiion to himself. B. walking away C. not letting bother him, 1D. changing styles in order to respond. 29. Acconding to passage information, « Noble would be most bothered by A. thoroughness, B. verbosity and fixation on details C.bluntuess 1D. asuccinet opposing argument GO ON TO THE NEXT PAGE. Test 2 zasat 44 © Examenackens 101 Passaces IN MCAT VeRsaL REASONING Passage VI (Questions 30-34) 10 6 2 35 45 50 Most moviegoers tend to sum up all of a film's features—acung, directing, spectal effects, and script—anto 2 blanket “I loved wt” or “hated 1” But movie industry ‘workers, and even film connoisseurs, can attest 10 the contribution of the movie's “cmemaucs.” or technical features, towards creating any movie's atmosphere. Arusuie movies are composed of a multitude of shows.” for discrete scenes usually lasting only 6 10 20 seconds, together, the hundreds of individual scenes combine to make up the movie For each shot, the director has many ‘options on how to film the same actors iteracting, and his choice has a great influence on how the audience pervenves the same action. For example, imagine that the movie's senpt calls for two actors to speak a fixed dialogue in a specified location Even while the director stays true to the scnpt, he has considerable leeway m how to film the scene. He may film an “extreme long shot” with the camera far away This tends to show the setting an penorama, ‘emphasizing the background whule underplayang the setors, ‘and 1S used primarily in outdoor scenes where the backdrop 1s particularly ampressive Or, he may employ the “long shot," which brings the camera close enough to capture the actors’ entire bodies, together wth some of the setuing This technique highlights the actors’ relation to their setung: it 1s useful for showing actors interacting with their setting ‘and 1s a Staple of action films The most common technique, the “medium shot” shows the actors from the waist up, ‘while incorporating a but of their setting, this shot focuses ‘peimanily on the characters rather than theirenvironment and 1s commonly used to draw atvention to importam dialogue Atthe nearer end of the spectrum 1s the “close-up,” where the camera 1s brought in close enough to focus on the ‘actors’ heads and faces. This has the effect of spotlighting ‘2 parncular actor, while hiding the setung and other actors It ean emphasize the actor's appearance or gestures and ‘can sometimes suggest the spotlighted character's isolation from his surroundings. There 1s also the “extreme close-u where the camera fixates on a single important or dramatic detail, such as the characters’ expressive eye gestures or fume bomb ticking towards zero Camera “angling” refers to the camera’. height from. the ground, and thus the vertical angle from which st (and the audience) views the action The most common angle 1s filmed at (adult) eye level, though some artiste films for or about children can capture a child's-eye view of the world by filming from a chuld’s eye level, looking up at most things Smilarly, even ordinary films can switch to “low angle” view by occasionally lowenng the camera to look upwards at a character or building The low-angle format suggests the object or characteris somehow Iaruer, grander, ddomanant, orntimidating In contrast, the “high-angle” shot positions the camera to “Jook down on” a character, which ‘ofien suggests he is nferior, powerless, weak, or mn trouble [Cepunght 8 2008 Exambracksr, ee ‘A “side-by-side” shot of two characters suggests they are 55 equal in smportance, while filmmg one character as seen over the shoulder of another emphasizes that character, while reminding audiences that he 1s being observed or heard, ‘Opties” refers to the spectal type of camera lens used tofilm each scene Like angling, tis 1s within the director's 60 purview, and durectors can switch cameras for each scene. ‘The normal lens attempts to duplicate the human eye's focus and perspective, and thus 1$ used for natural scenes and documentaries The special wide-angle lens has the artifictal ability to take an both the foreground and the background 65 simultaneously, but this exaggerated perspective makes on- screen objects seem farther away Its opposite, the telephoto Tens, can focus on only one plane while blurring others, it ‘makes objects appear closer and 1s often used sn conyunction ‘with the close-up. 70 “Perspective refers to the camera's ability to make an object seem closer or farther away. The special “soft-focus” mode films foreground objects normally, while blurnng the background into @ wash of color, This 1s often used in romantic pictures, fo emphasize lovers while evoking a beautful technicolor environment. The contrasting “deep 76 focus” mode 1s able to make near and far objects appear equally sharp; 11s used to film the foreground while not de-emphasizing the background, and is a staple of “getty” realise movies and documentanes 30, According to the passage, when deciding upon how to film a scene, one must take into account As how the audience 1s going to perverve the action B. the type of eamnera angles to be used. C. ifthe director will approve of the shot D. _afthe audience as going 10 ‘Tove sy of ‘hate wt” GO ON TO THE NEXT PAGE. a 32, According to the passage, a mature documentary would ‘most likely be filmed using A. anormal lens at “eye level B. a wide-angle lens in “decp focus” C. along shor’ in “deep focus” D. anormal lens 1n “deep focus” The passage discussion clearly suggests the hypothesis that one of the more important aspects mn the ‘making of a film 1s most A. figuring out what ‘most moviegoers’ are going 10 love éceiding how to make a movie “urtstic’ using a good director using a good cameraman por 20S Exe ~ereckers ne ‘VerBal REASONING Test 2 * 45 33. According 1© the passage, @ scene from a horror movie showing two lovers embracing, unaware of the huge monster closing in on them, would be filmed using A. an ‘eye level Tens B. a ‘hugh-angle' “long shot’ witha normal lens. C.__ a “low-angle” “long shot’ with a normal lens D. a ‘child’s-eye-level’ ‘close-up’ in deep focus “extreme close-up with a normal 34, Acconhing to the passage, a child’s film with three allemaung shots showing a mother scolding her small daughter, the daughter, and the father who 15 secretly listening would most likely be filmed using which sequence of camera angles, respectively? ‘A. ‘Low angle, ‘high angle, und ‘over the shoulder B. ‘Low angle, ‘Jow angle,’ and “high angle C. “High angle, “high angle, and ‘over the shoulder” D. “Over the shoulder,’ ‘high angle.’ and ‘Tow angle GO ON TO THE NEXT PAGE. Test 2 zusse 46 © Exanweaciers 107 Passaccs IN MCAT VERBAL REASONING Passage Vil (Questions 35~40) Under the “axes” theory of music appreciation, muste’s ‘appeal derives from the impression 1t leaves on hsteners, indexed with thesr understanding ofits technical composition and historical circumstances. Its indeed possible that both '5 factors coniribute to one’s evaluation, but probable that the ‘emotional impact far outweighs the technical execution in forming our final opinion of Consider the foundauons of Westeru music, sts beginniugs mn the Chnstian church of the Middle Ages 10 The scientific rules governing rts creation were not yet developed systematically, and there was Intle knowledge of the mechanisms of sound. Rules of consonance and dissonance were understood only narrowly, aud superstion mpeded the incorporation of clements such as polyphor 15 and the “devilish” tcxone Notation of music was sparse, with melodies wntten out without exact instructions on ‘rhythm oF pitch Working with the barest knowledge of acoustics, from laboriously copied texts, and doubtless ‘crude mstruments, the early proneers of church music strove 20 to portray the full glory of nothing less than the divine wath the full force of their emotion The extent to which they succeeded in proxtucing compelling 2 tnbute to the sunceruy of their emouon, f, even after a ‘mullenmum, can still compel admiration and feel 25 less-than-devout society Its truly timeless music iC 1S amazing, 1t 1s Tobe sure, medieval music was not without its esoteric academic conventions that were doubtless overlooked by ‘most medieval churchgoers. Yet the music cannot be said to have lost its eflechveness, because the composer did 30 not rely on the listener's understanding of the underlying ‘musical theory, but merely transmutied iis own reverence nto the work Since the Middle Ages, our understanding of sound and composition have increased exponentially to the pornt 35 where computers can not only synthesize but actually wnte ‘our music for us Yet this does not necessarily mean new music has improved With the scientific study of music theory, psychobiology, and acoustics, revealing whole new worlds of understanding. much ot today’s muste has 40 become obsessed with the minutest esoteric details; and sm us deve to become as intellectual and exact as possible, it hhas lost ts appeal to people Educated hsteners who studied principles of tWelve-tone composition may just grasp that there 1s a definite order at work 1n such pieces According 45 0 the “axes” theory, their antellectual sausfactioa at the mathematical correciness of serial muste should satisfy them as much as the spintuality of church muste, yet the former's unpopulanty suggests otherwise In fact, some serial muste radiates contempt for the 90 ustemer, it has consciously been made so complex as to be incomprehensible to the hstener, who 1s then made to tel Sapam © 2008 Examiackers, ne frustrated with hus own inabslity to understand the composer's intention. The med:eval-to-Renaissance composer covld be accused of dwarfing the listener in his exaltation of God, ‘58 with the excessive length and complexity of some pieces, but never of composing with the deliberate aim of alienating listeners, Returning to the “axes” theorem Of course reason 1s cone appropriate measure, for example, when, having studied 60 the purpose of a piece, one judges whether 1s emotional tone 1 appropriate to its stated aim (e.g., requiems should evoke gnef, ett.) Yet even in this context, a scems the “axes” model 1s a poor predictor of music’ effect since the emotional impact So outweighs the logical considerations, 65 and muste can be judged aesthetically even wsthout knowing, the curcumstances surrounding 1 35, Which of the following statements provides support for what the author probably means in satating that “even after a mallennium, st can stll compel admurauon and feeling ina less-than-devout society” (lines 23-25)? A. Mithons of people still flock to Egypt each year to admre the ageless wonder of the pyramids. B. Toursts still stand im awe of the Colosseum of Rome, though was the site of countless omble deaths ‘Though a majonty of Americans profesy a belief in ‘a supreme being, a minority actually attend any type ‘of religious service. ‘Young and old people alike are nereasingly tendin to purchase rap music with religious themes GO ON TO THE NEXT PAGE. 36. 37. ‘The author of the passage characterzes the “axes” theory ‘of music appreciation as one that A. can help accurately predict why listeners appreciate music or not B. ss completely erroneou C. may have been true an the past but ss now outdated D. 1s based upon a false assumpuon, ‘Suppose 1 could be established that almost all of the rmuste composed for the Chnstran church of the Muddles Ages could be deconstructed or simplified down to only ‘one or two consistent mathematical themes that seem to hold an almost universal appeal. The author of the passage ‘would be most likely to respond to this development by A. arguing the unversal appeal of any musical themes B. pointing out that the composers were unlikely to have been aware of this coincidence C.__ dispuung thatthe composers would have deliberately set out to alienate their Tisteners 1D. agreeing that these early composers were probably aware of universal musical theories If the author of the passage is nght that the “axes model” 15 a poor predictor of musie’s appeal, then follows that 200d" niusie can usvally be created following a set senies of guidelines B. music cannot be fully appreciated without some Knowledge of ts historical excurnstances. C. the emotional impression that a piece of music leaves om @ large group of listeners 1s unymportant D. the worth ofthe opinion ofa single, inghly educated musie ertue ts overrated Vera. REASONING TesT2 * 47 39. Regarding the composing o' strongly imphes that ‘A, alienating listeners way a deliberate goal B. tas comprehensible to the listener. C. composers are frustrated by Isteners’ insbihty © sal music,” the passage ‘comprehend D. ts more emotionally appealing to compose this way 40. If the author's primary entenon for yudging the “effectwveness” of muste way applied to manmade objects. which of the following creations would be most cffecuve” A. an huge underground particle accelerator B. the Alaskan pipeline CC. the Vietnam memorial D. the Suez Canal ‘STOP IF YOU FINISH BEFORE TIME IS CALLED, CHECK YOUR WORK YOU MAY GO BACK TO ANY QUESTION IN THIS TEST STOP. Test2 zasaL 48 © Exanesackens 101 PassaGes i MCAT Vensal REASONING 2008 Laamkrrkes la Verbal Reasoning Test 3 Time: 60 Minutes Questions 1-40 @ elses 50 © Exavnnacrees 101 Passaces iv MCAT Vera REASONING VERBAL REASONING DIRECTIONS: There are seven passages in this Verbal Reasoning test Each passage is followed by several questions After reading a passage, select the one best answer to each question If you are not certain of an answer, eliminate the alternatives that you know to be incorrect and then select an answer from the remaining alternatives. Indicate your selection by blackening the corresponding oval ‘on your answer document Passage | (Questions 1-6) 0 6 % 30 8 Imagine a Cartesian coordinate system of three- ‘damensional space with axes pointing up and down, left and night, and forward and backward Any point un this space ccan be descnbed by three terms the displacement 0 the Teft or right (x), the displacement forward and backward (9). and the vertical displacement (2) We can draw a line am this space between any {Wo pomnts, and the distance becween the two points 1s equal to the length of the hne However, life 1s animated, and just knowing the distance between two points cannot adequately represent animation Tn order to desenibe hfe, we must have a fourth axis Ifa living ere 1s traveling along our line, we can also ask, “At what ume 1s the creature at a given point?” That point in time must be ‘contiguous with two other points 19 time one preceding it and one following it ‘Tame (f) then 1s the fourth axis mn our space Lake the other axes, the tne axis 1s necessarily perpendicular to all fother axes so that that movement parallel co the time axis does not change the value of any coordinate but time Tn ‘other words, four bying ereature stands perfectly sullan our space, his x.y, and z coordinates remain constant, while his ¢ coordinate changes. It 1s unportant to note that & point with coordinates x, y, and 2 at one moment 1s not the same point 1m space as a point with the same x,y, and z coordinates a ‘moment later, hus changed. From this pomnt of view, a line parallel to the tame axis represents an object that Seems t0 sit perfectly sull Ennstean’s theory of relatwvitytellsusthat time and space are part ofthe same entity called the space-time continuum, In such a continuum, ume and space are indistnguishable Our previously described space 15 a model of this space- lume continuum. Einstein also theorized that light moves at ‘constant speed in a vacuum regardless of the perspective of the observer In our space-time continuum, a straght lune nearly perpendicular to the ume axis can represent the path and speed of a photon (a parucle of light) The slope with respect to the time axis gives the speed of light Since dus slope must be the same for all observers, observers that 200 Leamberekers, tne move in the x, y, of z direction will experience time more 40 slowly than those that remain still, Given thus information, 1t 18 concervable that time travel in the forward direction 1s possible Moving near the speed of light would slow time for the traveler, allowing him to arrive years in the future aiter what would seem hike 48 only moments to his body But what about going backward If space and time are truly indistimgurshable an the space-time continuum, backward time travel might seem as feasible as retracing your own footsteps through the space- ‘ume continu 50 The answer may ie not in the charactenstes of the space-time continuum, but 1n the phenomenon of light self Time may be an allusion of our perception of the space-time conbnuum, which seems to be intricately ed to the movement of light Photons themselves appear to us 55 to have a direction Light floods a dark room but cannot be absorbed to create darkness, Or can #0 An examination of the human sensory system reveals that we are not equipped 10 sense light retreating from us We may routinely move backwards in 60 time without bemg aware The space-time continuum may be a four-dimensional stone in which our past, present, and future 1s etched for us to continually relive each ume as af were the first Wich of the following asseruons 1s most clearly a thesis presented by the author? ‘A. Time travel is not possible B.A canis is necessary to desenibe Wfe C.Arraxis must be parallel to at Jeastone other axis un the mode! D, Humans are not equipped to sense light GO ON TO THE NEXT PAGE. 2. Lthe hypothesis of the passage § correct, one should find that a change only along the f axis A. would indicate photon movement. B. would indicate a flaw in the model C. would indicate an object that has traveled backwards D. would indicate an object that seems to sit pertecily stll 2. Which of the following sevennfic advances would most seriously challenge the hypothesis involving the “space: ‘ume continuum” (lines 28-30) Association of time and space characteristics Proof of Einstein's theory of relatrety Further correlation of the consistency of ight speed Confirmation of characterisucs distinguishing tume from space Pore 4. According tothe passage information, what would happeu if one were to travel near the speed of hight? ‘This person would remain perfectly stl ‘This person would travel back in time, Time would slow for this persou allowing him to travel to the future D. Time would avcelerate for ths person, allowing hum to travel into the future Comyn © 208 Eeamt: Versa REASONING TesT 3. #51 5. AF the author of the passage 1s right that the “space-time continuum may be a four-dimenstonal stone sn which our past, present, and future 1s etched for us to continually relive” (lines 60-63), then 1 follows that ‘A. the stone must he moving along the # axis B. atall umes we ure on the stone C. we have free will D. our hives are predetermined : 6 Which of the following statements 1s true concerning the fout-axes coordinate system described by the author” “yest 3 1. Knowing the distance between any ‘wo points wall adequately represent TH Allaxes are perpendicular t all other IIT Photons do not appear to us to have direction A. Lonly B. only €. Tand Monty D. Wand only GO ON TO THE NEXT PAGE. Asa 52 © Examuraceens 101 PassAces IN MCAT VERBAL REASONING Passage ll (Questions 7-13) The most obvious way for founders of charitable foundations to instruct _and control the trustees who administer the foundatton’s daily actwines 1s forthe founder to specify his intended aims for the foundation through 11s ‘5 charter documents These intentions can be enumerated affirmauvely (by outlming trustees’ expliet duties), or negatively (by listing those acts that the founder prohibits them from doing) ‘The more explicit the founder's instructions, the 10 less room there 1$ for trustee opportunism. At the same lume, explicit aims and prohibitions decrease the trustees’ fexibiiny, and an overly rigid set of imstructions may ‘cause the foundation to become obsolete or be dissolved. particularly m ight of changing circumstances For example, 15 af @ prosperous buggy-wiup manufacturer endowed a foundation that 1s only empowered to grant scholarships {o apprentice bugey-whip makers, the social impact of the foundation will dwindle as the mumber of apprentices falls Finally, when there ate no more such apprenuces. 20 the tounder’s heirs would be able to petition the courts 10 dissolve the foundation and assign its assets to them, Under the legal doctnne of ey pres (“close enough”), ung circumstances render a founder's stated purpose imposuble, illegal, or highly impractical, courts allow trustees to apply the foundation's endowment toward related chantable purposes. so long as they find that the founder's intent was to fund simular chartable projects, rather than the original project exclusively Where founders wish (© give guidance about their 30 recommended ams without the nsks of limiting the foundation’s usefulness or causing it to fal, they can attach “etter of guft” This letter—an expression ofthe founder’s ‘personal preferences—is not legally binding but can be used by trustees and cours to reinterpret the founder's probable 38 intent under changing circumstances ‘The foundanion documents also include provisions for removing trustees for misbehavior, either by specifying objective infractions (usually absenteeism) that require dismissal, or else by allowmg the board of trustees to 40 remove mdiidual members for cause by majorty vote However, oddly enough, the law usually denies the founder any right to ste trustces for fathng to enforce the terms of Ins foundation Of course, the founder wields the most power in his 46 smital selection of the omginal trustees. These orginal trustees are likely to consist of the founder himself, close fnend\family members. business associates, and personal Tawyers. Such agents are hkely to better understand—and respect—the founder's untent Copyrgh. 2 2008 Lxarkrarkers. ln 50 Once elected, though, most trustee boards are self- perpetuating Members are routinely re-elected until they ie oF reure. whew they do, the remaining trustees select a replcement There are methods (@ prevent trustee entrenchment Some foundations require term humts for 55 trustees or require retirement at a certain age This may be effective m provennag entreachment but 1s indiscriminate between capable and dishonest trustees. A less mechamcal arrangement 1s (0 require some “outside” trustees 10 be selected by dependent organzattons, corporations, ‘60 or political divisions. An even more rigorous system of accountability was devised by the Richardson Foundation allows donors . elect trustees annually, giving them votes proportional to their dollar contribution to the foundation for that year This leis those with the most invested in 65 the foundauon run xt, and probably encourages increased donation by founders seeking to take control of the board 7. A “letter of gift” (lune 32) would most hkely be used when ‘A. documentation of a donation was required. B. a donor wished to express a personal preference, CC. the guidance of a trustee way requested D. the founder wished to give some guidance GO ON TO THE NEXT PAGE. 8, Suppose a long-dead, affluent lover of birds had expressed his rather vaguely written desire that all of his momes go towards studying and protecting the now-extinet passenger pigeon Since this activity 18 no longer possible, the court ‘would most hikely A. fine that another similarly threatened anal spectes ‘would suffice since the activities are now impossible ‘and the founder was not explttt in his aums B, find that another similarly threatened bird species ‘wonld snffice since the founder loved all birds and ‘wats not expliet tu hus ams, C. find that the monies should revert to the hes since there 18 n0 other activity that 1 “elose enough, D. find that the monies shonld revert to the hetrs since the founder was explicit n his aims, which are now impossible, 9. According to the passage, “Some foundations requare term Inmuts for wustees or requute retirement at a certain age” (lines 54-55). However, this might be disadvantageous to the founder because A. this requirement does aot discriminate between ‘capable and dishonest trustees, and the founder ‘wields the most power 1n tus inital selection of the ‘origunal trustees, B, this requyement cannot be adequately enforced after the Founder 1s gone, and the founder wields the most power m his inital selection of the original trustees. C.__theoriginal rusts tha the founder ehose may refuse this short-term occupation, and the requirement 10 renre cannot be enforced D. the Jaw usually denies the founder any nght 10 sue trustees for faihng to enforce the terms of his foundatiou, and the requirement to retre cannot be enforced 10, Assume that the wealthy buggy-whip manufacturer's, heirs would prefer to inhent her wealth rather than see i donated to another ‘chartable purpose” where uone of them 1s employed as one of the well-paid trustees Wiich of the following hypotheses does this assumption suggest? A. The trustees would be disappointed by a judge's cy pres raling B. The heirs would go to court iw au attempt to obtain a cy pres ruling, GC. Acy pres ruling would satisfy both the trustees and the hews D. The heirs would be dismayed by a judge's cy pres ruling Copyrighr © 2008 Eva Vero Reasoning Test 3 * 53 11. What assumption is implicit n the sdea that “an even ‘more rigorons system of accountability ets those with the most invested in the foundation run it, and probably encourages increased donation by foundery seeking to take control of the board” (hines 60-66)? A. That those donors who have more money to invest wall not want 10 take contro! of the board. B. That those donors who seek to take control of the board wil be more accountable C. That donors are motivated by accountabihty. D. That those who have the most anvested 1m the Foundation are motivated by founders seeking 10 take control of the board 12. According to the passage, the “founder wields the most power iu his mitial selection ofthe original trustees” (lanes 44-45) forall of the following reasons 1 All the trustees are probably going to be personally known by the founder TW The founder ss bkely to appomnt himself asa trustee TIL ‘The intent ofthe founder s most likely to be respected by these trustees. A. Honky B. Land Monly C. and Ut only D. (.lland ur 13, The passage suggests that 11s author would probably dvsagree with which of the following statements? A. There are advantages and disadvantages 1m very ‘explicit mstructions by the fouuder B. Very explicit insiructious by the fouuder are invariably dhsadvantageou C._ Very explie nstructions by the founder may cause the foundation to become obsolete D. Heirs would have a greater chance of dissolving @ foundation based on overly rigid instructions GO ON TO THE NEXT PAGE. 54 © Exanncracrens 101 Passaces in MCAT VeRoAL REASONI Passage Ill (Questions 1 10 6 20 as 35 6 ‘0 19) Sigmund Freud, the “Father of Psychotherapy,” soften eniuewed by modern psychology for the fanciful nature of his theories. which do not seem to have any verifiable basis an either psychological experimentation or normal people’s conscious thoughts. For example, the “Oedipal complex” (which supposedly drives all men to desire to kill their father in order to mate wath their mother) seems to appear only in a single Greek myth rather than an real life Correctly, Amencan psychotherapy has targely rejected Freud's theories, but his lively and mvenuve storytelling abulity ensures that his anfluence lingers on an the popular uunderstandug, with serous implications for women Freud's famous remark that “anatomy 1s destiny” means that a woman's anatomical “limutations” (the vagma 1s seen 8 a lack of a pemss, supposedly a cause of “penal envy”) doom her to contribute litle towards crvihization’s material progress In Calicanon and us Discontents, Freud sums up ewilvation largely im terms of increasing technology, Jessure, and protecuon from nature Accordingly, he states that the tarng of fie was the firs step to ewthzation But then, Freud invents a bizarre theory of why it had to be a ‘man who first tamed fire men, he says, view flame as aval phallus and snstinctwvely desire 10 extinguish 1t by unnabing fon it, which gives them the subconscious impression of winning & quasi homosexual competiuon Under Freud's theories, m order for a person to create contnbutions to civilization, “sublimation” must occur That 1s, the instinct toward sexual graufication mustbe suppressed, so that sexual energy can be channeled into matenally produetuve work TThe male alone. because of hns pemss’s long-range urinating capability, had the ability to put out the fire Thus ability ‘was unimportant in self, except that € carmed wath st the simultaneous possibilty of ror uulzing that ability ("self suppression”), which would make sublimation possible Women, because they could not fulfill the isunct, could ‘not suppress or sublimate ateither, so they were assigned the role of guardian of « hearth-fame that they were powerless toextinguish Freud's presentation of lus theory deliberately complicates tracing the “role” given to women back fo any ppasueular agent, so thet their inferior role 1s meant be seen as natural and mmutable What imphicauions does this have for women? It seems directly targeted at them, smce w presents 20 lumstations for men in creating ewvilzation, but ascribes to women a necessanly lower potential for sublumation- induced acinevements, such as art and science ‘The female's supposed protecuveness towards the fie thatthe male wants to extinguish, and the resulung competition of the mate's ambition against the female's possessiveness predicts a natural antagonism between the sexes Furthermore, this perpetuates the common ascription of passivity to women and actvity to men, a mote that Freud finds unpalatable in Copjnghtt@ 2008 Exarkrarkery he ‘other works but overlooks ins own The results of women's 55 lumts being attnbuted to a natural ynequahty are that any differences in the socral status of women are (a) made to seem beyond correction, (b) made to be beyond gnevance, since the agent 1s nature rtself, and (c) permanent These combine to make Freud’s role for women a true “destny: {60 an that st1s made to seem mrevocable Feminist theonsts, ike Srmone de Beauvior, have made some inroads against the influence of Freudians. While de Beauvior concedes that women might be ataslight biological disadvantage by the demands and dangers of pregnancy 65 and menstruation, she argues that this slight difference as msuflicient to account for the completely mnfentor role assigned to women by social constructions Her Second Sex progresses from “biology” to the greater influence of societal constructions The book rejects brological theories 70 as brased by sexual polines, noting that Freud invented a “sexual” anatomy separate from biological anatomy, and practiced a type of biology that labels female anatomy with ‘2 bias toward controlling it by presenting women’s bodies aseither mcomplete, inaetve, or weak, and thus im need of 78 mento give meaning to their existence Faced with a biology that merely reaffirms social preyudiees, de Beauvior rejects a as an objecuve science and explores societal prejudices directly This 18 what she me: he says, “One 18 not born, oue becomes, a woman” 14, According to the author, which of the following 18 most Lukely t be (rue about the relauonshup between the “fire” and “women”? A. Freud proved that women were protective towards the hearth-fire B, There 1S no relationshup between women and the fire C.-Men wall continually put out the fire by urinatung on a D. Women have been assigned the role of guardian of the heath GO ON TO THE NEXT PAGE.

You might also like